VetPrep Flashcards Preview

Clinics! > VetPrep > Flashcards

Flashcards in VetPrep Deck (216)
Loading flashcards...
1
Q

appetite stimulants

A

cycloheptadine, diazepam, mirtazipine, oxazepam

2
Q

bandage for coxofemoral joint luxation

A

ehmer sling

3
Q

5yo Dog hx chewing on tennis balls and frisbees. oral exam had brown, worn-down incisors, canines, and premolars. Otherwise healthy.
Why are they discolored?

A

formation of tertiary dentin, which stains easily.

4
Q

A 1-year old female spayed Doberman Pinscher has presented after being hit by a car. Initial chest radiographs show mild contusions, and the patient appears to be otherwise stable. A right mid-shaft long oblique femoral fracture has been identified. Routine pre-operative blood work is unremarkable. A buccal mucosal bleeding test (BMBT) is elevated at 6 minutes. What will you administer prior to surgery?

a. desmopressin acetate
b. vitamin k
c. whole blood
d. 1,25 dihydrocholecalciferol

A

a. desmopressin acetate

The correct answer is desmopressin acetate (DDAVP). Administration of desmopressin results in release of von Willebrand factor, which will help this patient with clotting. Given this dog’s breed and elevated BMBT there is a very strong likelihood she is afflicted with von Willebrand’s disease. In Dobermans this results in an inability to form a clot. This can be life threatening if the dog is taken to surgery.

A whole blood transfusion does not provide an adequate source of von Willebrand factor but may be necessary if the patient’s bleeding cannot be controlled despite appropriate pre-operative measures. 1,25 dihydrocholecalciferol is the active form of vitamin D which aids intestinal resorption of calcium. The BMBT does not assess factors 2, 7, 9, or 10 and therefore vitamin K is not indicated.

5
Q

6mo labrador limping and not gaining weight. In an outdoor kennel with other hunting dogs. QAR, BCS 3/9. Pale MM, CRT 2s. Front and LH paw pads chewed at frequently, visibly ulcerated.
Diagnostic test?
[a. cbc/chem b. Rads c. Biopsy d. skin scrape e. fecal float f. PCV/TP]

A

e. fecal float
hookworms cause anemia, enteritis, coughing during larval migration, and dermatitis. Any young dog that is failing to thrive/pale mm should be tested for intestinal parasites. Hookworm dermatitis (Ancyclostomiasis) is typically seen in conditions with poor sanitation.

6
Q

HBC
HCO3-12 mmol/L (17-24 mmol/L)
Total CO2-14 mmol/L (14-26 mmol/L)
Lactate-2.3 mmol/L (0.5-2.0mmol/L).

Assessment? [a. resp acidosis b. resp alkalosis c. met alkalosis d. met acidosis]

A

d. Metabolic Acidosis

low bicarbonate and total CO2 level is consistent with a metabolic acidosis.
Lactate above 2.0 mmol/L implies that there is anaerobic metabolism occurring in this patient as a result of inadequate tissue perfusion, perhaps due to bleeding and trauma.

Nothing can be said about respiratory acidosis or alkalosis because no PaCO2 or pH is provided.

7
Q

A 6-month old Labrador Retriever presents for regurgitation. The dog is otherwise happy and doing well, according to the owner. A chest rads = megaesophagus. What therapeutic intervention is most important in the management of this dog’s condition?
[a. terbutaline b. small frequent meals c. tensilon (edrophonium) d. strict cage rest e. glucocorticoids]

A

b. small, frequent elevated feedings.
megaesophagus. Causes of include congenital(It is 6months), vascular ring anomaly, endocrinopathy, secondary to neuromuscular disease such as myasthenia gravis, or secondary to esophagitis. Affected animals should be fed a high-calorie diet, in small frequent feedings, from an elevated or upright position to utilize gravity in emptying the flaccid esophagus. Pulmonary infections due to aspiration pneumonia should be identified and treated appropriately. Anti-cholinesterases can be tried, especially in cases where myasthenia gravis is the underlying cause, but edrophonium is too short-acting to be prescribed for chronic use, so you would probably choose pyridostigmine. Terbutaline is a bronchodilator and is not indicated in this case. Strict rest would not be particularly helpful in this case.

8
Q

A 2-year old male castrated Bull Mastiff with a history of travel in the west coast presents to your clinic for a chronic cough and a recent development of lameness of his right hind limb. On physical exam you notice a draining lesion over the lame region of the right hind limb. You perform chest radiographs and see a diffuse bronchointerstitial pattern which is nodular in some regions. You also identify hilar lymphadenopathy. You suspect that you know what you are dealing with and perform a broncho-alveolar lavage for cytology. Just as you suspected, you see spherules. What is your diagnosis?
[blastomycosis, aspergillosis, cryptococcus, histoplasmosis, coccidiomycosis]

A

The correct answer is Coccidioidomycosis. The travel history and clinical signs are consistent with this answer. Additionally, finding spherules on cytology is pathognomonic for Coccidioidomycosis. Prolonged antifungal treatment will be necessary. Fluconazole is the treatment of choice. Ketoconazole and itraconazole are good choices. With blastomycosis you see broad based budding of the yeast. With Cryptococcus neoformans you will see narrow-based budding.

9
Q

A dog presents for an acute onset of right hind limb lameness. On physical exam you note the limb to be slightly externally rotated and shorter than the contralateral limb. You suspect a coxofemoral luxation. What is the most likely direction of luxation?

A

craniodorsal

10
Q

What is a potential side effect of administering diethylstilbesterol in an incontinent bitch?
[a. hemolysis, b. hemorrhagic diarrhea, c. KCS, d. bone marrow suppression]

A

The correct answer is bone marrow suppression. This is a hormone, and like many other hormones, can result in bone marrow suppression. This is one of the reasons veterinarians choose to use phenylpropanolamine for urinary incontinence. This drug is a weak alpha agonist and works on the muscles of the urethral which results in increased sphincter tone. KCS is more likely to occur with sulfa containing drugs. Most drugs can cause some sort of diarrhea as a side effect, but rarely will it be hemorrhagic.

11
Q

Which of the following are effects of non-steroidal anti-inflammatory drugs that are non-selective COX inhibitors?

a. reduction swelling, neutrophil inhibition
b. GI ulcer, inhib platelet, analgesia
c. inhib inflam, dest platelets, red. fever
d. GI ulcer, platelet destruction, inhib neutrophils

A

B
Non-selective COX inhibitor side effects include GI ulcers, inhibition of platelet function, renal damage, and reduction of fever. NSAIDs reduce inflammation and pain by blocking the COX-1 and COX-2 pathways.

12
Q

Two months ago, you prescribed one of your feline patients a course of pradofloxacin (Veraflox), a fluoroquinolone antibiotic, to treat an abscess. The owner of the cat also owns a small dog who now has pyoderma with Staphylococcus susceptible to fluoroquinolones. She asks you if she can use the remaining pradofloxacin for her dog that she has left over from her cat. What should you tell her?

A

Pradofloxacin (Veraflox) is an orally administered, liquid fluoroquinolone antibiotic that was FDA approved in 2012 for use in cats to treat skin infections (wounds and abscesses) caused by Pasteurella multocida, Streptococcus canis, Staphylococcus aureus, Staphylococcus felis, and Staphylococcus pseudointermedius. The drug is not for use in dogs and it has been shown to cause bone marrow suppression resulting in severe thrombocytopenia and neutropenia.

Enrofloxacin has been associated with retinal degeneration in cats, particularly at high doses and is used with caution for this reason.

13
Q

What is the most likely complication associated with the post-anesthetic recovery of the dog in the photograph receiving Clotrimazole? (gel applied IN with Qtips)
[a. bacterial rhinitis b. asp pneumonia c. laryngeal inflammation d. seizure

A

c. The correct answer is laryngeal inflammation. Seizure is a possible sequela if the cribriform plate is compromised. If this were the case, the complication would likely be fatal. With severe swelling, the administration of corticosteroids and re-intubation is indicated. Sometimes these dogs may need to be intubated for days.

14
Q

By far, the most likely diagnosis is intervertebral disc disease (IVDD) based on the history and myelogram showing a collapsed intervertebral space at T12-13 and an extradural compressive lesion originating from the same site.

Most important aspect of MEDICAL MANAGEMENT of this condition?

A

IVDD can be managed by surgical decompression or non-surgically. The most important aspect of non-surgical management of IVDD is STRICT cage rest, usually for a full 8 weeks.

Exercise should be avoided during the period of cage rest. Steroids, analgesics, and/or muscle relaxants may be useful additions to the management of these dogs.

15
Q

Medical management of pyometra?

A

PGF2a
Although ovariohysterectomy is the treatment of choice for dogs with pyometra because it is curative and preventative for recurrence, medical management of pyometra can be considered in dogs of appropriate breeding age that are reproductively valuable and free of life threatening complications including septicemia, endotoxemia, or organ dysfunction. Options include prostaglandins to induce regression of corpora lutea, which relaxes the cervix and stimulates myometrial contractions, promoting expulsion of the uterine contents. Additional options include dopamine agonists (such as cabergoline and bromocriptine), which act to inhibit prolactin production from the pituitary gland. Patients should be monitored frequently during and after medical management of pyometra and should improve within 2-4 days. Successfully treated bitches should be bred during their next cycle after treatment.

16
Q

A 5-year old female Golden Retriever presents for lethargy, dark runny stool, and dehydration. Bloodwork from yesterday showed markedly elevated plasma endogenous ACTH levels, K+ = 6.2 (3.9-5.1 mEq/L), Na+ = 135 (142-152 mEq/L), BUN = 62 mg/dl (8-28 mg/dl). Long-term maintenance therapy for this patient should most likely include:
A. physiologic pred
B. Min,glucocorticoid, NaCl
C. fluids and daily IV dexamethasone sodium phosphate
D. Mineralocorticoid only

A

B
The correct answer is mineralocorticoid, glucocorticoid, and NaCl supplementation. This dog has hypoadrenocorticism which is usually a deficiency in both glucocorticoids and mineralocorticoids. Elevated endogenous plasma ACTH concentrations with hypoadrenocorticism means the disease is due to primary adrenocortical insufficiency and failure to produce both types of corticosteroids (as opposed to secondary adrenocortical insufficiency, when the pituitary does not produce sufficient ACTH). Maintenance treatment of this disease includes supplementation with mineralocorticoids (e.g. Fludrocortisone acetate), glucocorticoids (e.g. Prednisone), and sodium chloride in the diet. IV fluids and dexamethasone sodium phosphate is used in an acute crisis.

17
Q

A 6-month old male intact Weimaraner presents for the right forelimb swelling seen in the picture. He is also pyrexic at 104 F (40 C) and reluctant to walk. His litter-mate had the same clinical signs which resolved with a course of prednisone. What should you ask the owners to support the diagnosis you suspect?

a. hx of trauma?
b. contact with snakes/spiders/wildlife?
c. travel hx?
d. vaccinated recently?

A

D.
This puppy likely has hypertrophic osteodystrophy (HOD). Weimaraners are predisposed and litter-mates will commonly be affected. Although the exact cause is unknown, the leading hypothesis is recent vaccination leading to hyper-reactivity of the immune system. In a study of 53 Weimaraners with HOD, all had been vaccinated within the past 30 days.

18
Q

Which is important in managing struvite urolithiasis?

a. inc. dietary protein
b. USG <1.020
c. urine pH 8-8.5
d. inc. Mg in diet

A

B
The correct answer is maintaining urine specific gravity <1.020. An important principle in management of any patient with urolithiasis is to maintain dilute urine so that solutes do not concentrate to the degree that they form precipitates that lead to stone formation. Struvite stones (magnesium ammonium phosphate) tend to form in alkaline urine so ideal urine pH for struvite stone formers is usually around 6.5. Other principles of management are to decrease urea in the urine, often done by moderate protein restriction and to decrease the minerals involved (magnesium and phosphate). Finally, a key element in struvite stone formation is the presence of a urinary tract infection, particularly with urease-producing microbes, most notably staphylococcus and proteus. Monitoring, preventing, and treating UTIs is an important aspect of management of these patients.

19
Q

Clinical finding in lymphangiectasia?

a. hypercholesterol
b. lymphocytosis
c. hypercalcemia
d. panhypoproteinemia

A

d
The correct answer is panhypoproteinemia. Lymphangiectasia is a classic example of a protein-losing enteropathy. It is characterized by dilation and dysfunction of intestinal lymphatics and leakage of protein-rich lymph into the intestinal lumen. As a result, protein, cholesterol, and lymphocytes are all lost. Calcium is also frequently low due to either low albumin or vitamin D and calcium malabsorption.

20
Q

T or F

Treatment of localized Demodex in young dogs is usually unnecessary. It usually clears up on its own.

A

True

21
Q

progressive muscle loss on the right side of the face. Differential?

A

Trigeminal or Facial neuropathy.

The correct answer is right trigeminal neuropathy. Hopefully, you were able to identify the presence of significant muscle atrophy of the right masseter and temporalis musculature. The nerve innervating these muscles is the trigeminal nerve. Recall that the trigeminal nerve (cranial nerve V) has primarily a sensory function, with the exception of innervating the muscles of mastication. The facial nerve (CN VII) innervates the muscles of facial expression and is the main motor nerve innervating the face but is not responsible for the muscles that are atrophied in this case. Masticatory muscle myositis is an autoimmune condition that chronically leads to atrophy of the same muscle group, but what differentiates this case is that the signs are unilateral. Idiopathic polyradiculoneuritis is the medical term for a condition sometimes referred to as Coonhound paralysis, which is a condition that diffusely affects all motor nerves.

22
Q

VPCs tx?

a. oral mexiletine and recheck
b. IV furosemide
c. no tx
d. bolus lidocaine

A

There are several general guidelines/recommendations regarding treatment of ventricular premature contractions which is what is shown on the ECG strip. These include a heart rate of over 180 bpm, pulse deficits, clinical signs, and VPCs for over 20 seconds in duration. In this case, the heart rate is visible on the recording and clearly not elevated. The patient’s physical exam findings are not suggestive of an immediate problem secondary to the arrhythmia. There are many dogs with intermittent VPCs that are not of clinical importance. However, it is recommended that the patient be evaluated regularly and ideally have a consultation with a cardiologist to ensure the progression of heart disease is being minimized.

Furosemide is not indicated as there is not an indication that the patient is fluid overloaded or in fulminant heart failure.

Mexiletine is a Class IB anti-arrhythmic and is not necessary given the lack of severity in the ECG findings.

23
Q

Most common cause of pyometra in dogs over 8 years old?

a. trauma
b. ovarian neoplasia
c. pregnancy
d. cystic endometrial hyperplasia

A

D. The correct answer is cystic endometrial hyperplasia (CEH). Older intact females develop CEH from repeated exposure to progesterone. Progressive thickening occurs and glands become hypertrophied. This sets up the uterus for development of pyometra.

24
Q

Signs of chocolate toxicity?

A

The toxic principle of chocolate is methylxanthines (specifically theobromine and caffeine). Methylxanthines can cause CNS excitation tachycardia, and vasoconstriction. Signs include vomiting, diarrhea, hyperactivity, polyuria, polydipsia, lethargy, tachycardia, cardiac arrhythmias, seizures, and death.

25
Q

Mitral regurgitation in dogs eventually results in which of the following?

a. Concentric hypertrophy of the left atrium and eccentric hypertrophy of the left ventricle
b. Eccentric hypertrophy of the left atrium and concentric hypertrophy of the left ventricle
c. Eccentric hypertrophy of the left atrium and left ventricle Correct Answer
d. Concentric hypertrophy of the left atrium and left ventricle

A

C. The correct answer is eccentric hypertrophy of the left atrium and left ventricle. Mitral regurgitation is the process of blood flowing back from the left ventricle to the left atrium due to a defect in the valve. The excess blood flow causes volume overload of the left atrium leading to eccentric hypertrophy or dilation of the chamber. The excess blood volume will then return to the left ventricle during the next diastole, resulting in dilation and eccentric hypertrophy of the left ventricle as well. Concentric hypertrophy occurs in cardiac chambers when they are pressure overloaded, such as in the case of pulmonic stenosis or aortic stenosis.

26
Q
4y MN Pit bull, may have gotten rodenticide 2 hours ago. induce emesis = green vomit. 
What do you recommend next?
a. just monitor
b. vitamin k 6 wk
c. check serum Ca
d. hospitalize and treat any neuro signs
e. vit e and selenium
A

The bright green vomit confirms the owner’s suspicion of rodenticide ingestion because many rat poisons contain a bright green dye. Dogs that ingest these products may have bright green vomit or stool. Brodifacoum is a vitamin K antagonist commonly used in rodenticides. Ingestion of this compound causes hemorrhaging after several days due to a lack of production of new clotting factors. Treatment for this condition requires vitamin K1 administration for 4-6 weeks.

27
Q

clinical signs of ancyclostoma in dogs and humans?

A

dogs - anemia

humans - cutaneous larval migrans

28
Q

You are examining a dog for lameness and on radiographs; you find hypertrophic osteopathy of the metacarpal bones. What is the likely cause of this lesion?

a. Ca def
b. bone tumor
c. pulmonary neoplasia
d. vit d def

A

c. The correct answer is pulmonary neoplasia. Hypertrophic osteopathy is visible as periosteal proliferation in the diaphyses of affected bones. Usually the metacarpal and metatarsal bones are affected first, and it may progress to the long bones. Lameness, pain, and swelling are usually evident. Joints are unaffected. The cause is usually primary or metastatic pulmonary neoplasia, although other diseases in the thorax including bronchopneumonia, Spirocerca infection, or congestive heart failure, may also be associated. Less commonly, abdominal neoplasia may be seen with this condition.

29
Q

primary goal in treating diabetic ketoacidotic dog?

A

The correct answer is correction of acidosis, electrolyte abnormalities, and hyperosmolality. Initial treatment of DKA patients is aimed at correcting acidosis by administering bicarbonate (if the total bicarbonate is markedly low), correcting electrolyte abnormalities with IV fluids, potassium and phosphorus supplementation, and correction of hyperosmolality. Regulating blood glucose, getting the animal to eat, controlling obesity and concurrent disease should be addressed after immediate life-threatening issues are resolved.

30
Q

2 month female maltese. L heart base grade 5 murmur. Enlarged L heart and pulmonary vasculature.

diagnosis?

A

The correct answer is patent ductus arteriosus. PDAs results from the failure of ductus closure after birth resulting in a shunt (usually left-to-right) from the aorta to the pulmonary artery. The shunting causes excess volume in the pulmonary arteries, veins, left atrium, left ventricle, and aortic arch. The signalment of the dog and described clinical features and diagnostic findings are classic for PDA. In addition to the enlarged left heart and pulmonary vasculature, there is also usually an aneurysmal bulge of the aorta near the PDA on the DV view. The bounding pulses result from a widened pulse pressure (difference between systolic and diastolic pressure) because of the run-off of blood from the aorta to the pulmonary artery. PDAs are the most common congenital heart defect in dogs. The most common congenital heart defect in other domestic species is ventricular septal defect.

31
Q

What test should be run to monitor the efficacy of treatment for an Addisonian dog being treated with supplemental mineralocorticoids and glucocorticoids?

A

The correct answer is serum sodium and potassium. Addisonian patients deficient in mineralocorticoids will usually exhibit the classic findings of hyponatremia and hyperkalemia. Monitoring serum sodium and potassium concentration while treating with exogenous mineralocorticoids is the best way to determine if the level and frequency of dosage is adequate for the patient.

An ACTH stimulation test can be used to monitor the efficacy of treatment of a patient with Cushing’s disease, but does nothing for an Addisonian.

32
Q

A 6 year previously healthy old dog presents in acute respiratory distress. On physical exam, you auscult a split second heart sound. You take neck and thoracic radiographs which appear normal. What is the most likely cause of this dog’s dyspnea?

A

The correct answer is PTE. PTE should be immediately suspected in any dog with profound dyspnea and unremarkable radiographs. The split second heart sound is heard due to pulmonary hypertension from the PTE. This is not congestive heart failure because the heart is not enlarged, and there is no pulmonary edema on the radiographs. Aspiration pneumonia is less likely because of the history and normal lung films. With pneumothorax, you would expect decreased dorsal lung sound and consistent radiographic changes.

33
Q

A 4-year old male castrated Schnauzer presents after being rescued from a house fire. On presentation, the dog has a respiratory rate of 45 breaths per minute with increased inspiratory effort and moderate distress. The dog’s mucous membranes are bright red. lateral rad shows interstitial pulmonary pattern.

What is the likely cause of the bright red color of the dog’s mucous membranes?

A

The bright red mucous membranes in a dog suspected of having smoke inhalation are concerning for carbon monoxide toxicity. Carbon monoxide displaces oxygen on hemoglobin molecules, forming carboxyhemoglobin complexes, which leads to tissue hypoxia.

The dog should be supplemented with 100% oxygen which reduces the half-life of the carboxyhemoglobin complexes from 4 hours on room air to 30 minutes. Other treatments for smoke inhalation include humidifying the air to promote mucociliary clearance and to prevent drying injuries to the airways.

34
Q

When measuring Vitamin B12 (Cobalamin) and Folate levels in a dog with suspected small intestinal bacterial overgrowth, which finding is most supportive of this diagnosis?

A

The correct answer is increased folate, decreased cobalamin. The reason for this is that bacteria synthesize folate leading to decreased absorption of cobalamin.

35
Q

Which of the following is most suggestive of pyelonephritis?

a. bacterial or WBC casts in urine
b. pain on abd palpation
c. small irregular kidneys
d. bact/WBCs in urine

A

A. The correct answer is bacterial or white blood cell casts in urine. Bacterial or white blood cell casts are highly suggestive of a past or present bacterial infection in the kidneys. Bacteria or white blood cells in the urine may be due to cystitis. Small, irregular kidneys may be due to any chronic pathology in the kidneys. Pain on abdominal palpation can be caused by pain from any abdominal organ or structures adjacent to the abdomen. Other signs associated with pyelonephritis include fever, anorexia, depression, and vomiting. A nephropyelogram may show blunted, dilated calices and dilated, tortuous ureters. Renal biopsy and culture would give the definitive diagnosis of pyelonephritis.

36
Q

A 6-month old male German Shepherd Dog presents for castration. A 4/6 left systolic murmur is heard best over the apex of the heart. Thoracic radiographs reveal a moderately enlarged left atrium. What is the most likely diagnosis?

A

The correct answer is mitral valve dysplasia. This abnormality is a congenital anomaly of the mitral valve. The valve leaflets may be thickened, fused, fibrosed, etc. The chordae tendinae or the papillary muscles attaching to the mitral valve may be irregular as well. German Shepherd Dogs, Great Danes, and other large breed dogs are predisposed to this disease. Myxomatous mitral valve degeneration occurs in older dogs, but would otherwise result in similar physical exam and radiographic findings as those described. Tricuspid dysplasia results in an enlarged right heart, and the murmur would be heard best on the right side of the dog. Aortic stenosis results in concentric hypertrophy of the left heart, which may or may not be detectable on radiographs. Also, the murmur is heard best at the left heart base with aortic stenosis.

37
Q

A 10-year old Shih Tzu presents for collapse. A diagnostic work up shows a mass on the right kidney, which was subsequently removed and biopsied as renal carcinoma. Which one of the following can be associated with renal carcinoma?

a. hyperestrogen
b. hyperCa
c. Hyperadrenocorticism
d. thrombocytopathy
e. polycythemia

A

E. Renal carcinomas have been associated with erythropoietin secretion, causing elevations in hematocrit as a paraneoplastic syndrome.

Hypercalcemia is associated with anal sac adenocarcinomas, lymphomas and other tumors.
Thrombocytopathy can be seen with hemangiosarcoma and multiple myeloma.
Hyperestrogenism can be seen with Sertoli cell tumors.
Hyperadrenocorticism is caused by functional adrenal or pituitary tumors.

38
Q

Approved medical tx for oral(metastatic) melanoma?

a. doxorubicin
b. ONCEPT vax
c. Palladia
d. vincristine
e. mitoxantrone

A

The ONCEPT vaccine, also commonly referred to as the “melanoma vaccine” was given a conditional product license in 2007 and was USDA approved in 2010 as a therapeutic vaccine for cancer treatment.

Palladia is a receptor tyrosine kinase inhibitor approved for use in mast cell tumors in dogs. Doxorubicin, vincristine and mitoxantrone are human chemotherapy drugs commonly used off-label in veterinary oncology but not routinely for melanoma.

39
Q

A 2-year old Siberian Husky presents for crusting and scaling of his foot pads, periorbitally, and along his mucocutaneous junctions. Based on the breed and clinical signs, which supplement should be added to this dog’s diet?

a. vit e
b. Zn
c. Ca
d. Cu

A

The correct answer is zinc. Huskies are predisposed to zinc-responsive dermatosis in which individuals have a higher requirement for zinc in their diets due to a defect in absorption of the element. Clinical signs include crusting and scaling of foot pads, elbows, eyes, and mucocutaneous junctions.

40
Q

A 6-year old female Saint Bernard presents for an acute onset of restlessness, retching, and hypersalivation. On physical exam you notice abdominal distension, tachycardia, a prolonged capillary refill time, and pale mucous membranes. What is your most likely diagnosis?

A

GDV
The correct answer is gastric dilatation and volvulus. To answer this question correctly, you need to pay attention to the signalment. In this case, you have a deep-chested large breed dog. These dogs are predisposed to GDVs. Additionally, the history of acute onset and clinical signs are consistent with GDV. You can immediately eliminate parvovirus by looking at the age of the dog. For the most part, parvovirus is a disease of puppies. Gastric foreign body and pancreatitis are good differentials; however, the abdominal distension seen on physical exam should help you lean towards GDV. Surgical treatment would involve a gastropexy to anchor the stomach in place so that a GDV does not recur.

41
Q

Which of these is a major potential adverse side effect of the chemotherapeutic drug cyclophosphamide in a dog?

A

The correct answer is cystitis. In addition to the usual side effects of chemotherapeutic drugs such as myelosuppression and GI side effects, a major concern with cyclophosphamide administration is causing sterile hemorrhagic cystitis due to a breakdown product of the drug called acrolein. To reduce this risk, animals should be encouraged to drink lots of water or be given fluids and encouraged to urinate frequently to prevent the toxic compound from sitting in the bladder for a long time. The drug that causes cardiotoxicity is doxorubicin. The drug that causes ileus is vincristine. The drug that causes anaphylaxis is L-asparaginase. The drug that causes nephrotoxicity is cisplatin.

42
Q

Which of these is least likely to cause respiratory distress and an increase in respiratory rate in animals that are victims of smoke inhalation (as in smoke from a fire, not a cigarette)?

a. Co2
b. thermal damage to bronchi
c. laryngeal edema
d. CO

A

The correct answer is thermal damage to the bronchi. Carbon monoxide inhalation is an important component of smoke inhalation morbidity as it leads to carboxyhemoglobin formation and displacement of oxygen. Carbon dioxide inhalation at high levels leads to severe acidosis, which leads to an increase in respiratory rate. Damage to the larynx from the heat causes edema and swelling, contributing to upper airway obstruction. The heat damage does not usually reach the level of the bronchi or lower airways. Smoke also can inhibit pulmonary macrophage function, and skin burns can worsen pulmonary status.

43
Q

A new client from North Carolina comes to your practice to bring in her 5 year-old male neutered hound dog, Bart, and her 10 year-old male neutered domestic short hair, Spaz, for examination. Spaz has been doing fine and just needs his yearly preventative care, but Bart has been lethargic with weight loss and intermittent lameness. On physical exam, you note that the dog has a thin body condition score of 3/9, generalized lymphadenopathy and a fever of 104.2 F (40.1 C). The cat seems completely normal and healthy enough for vaccinations. During your attempt to vaccinate Spaz, he gets very agitated and scratches the owner on the arm. She notes that Spaz is kind of a difficult cat and that he has scratched her many times before; after one incident, the owner needed antibiotics for an infection that spread to her armpit from one of the cat’s scratches. She mentions that she probably should have told you that Spaz can be difficult prior to the exam. Given the dog’s (and owner’s) clinical signs and travel history, what disease might you suspect at this time?

a. toxoplasmosis
b. lyme
c. bartonella
d. ehrlichia
e. babesia

A

Bartonella henselae is the bacterium that causes cat-scratch disease. In cat-scratch disease, the owner is infected with the bacteria from flea feces when the cat bites or scratches the owner. Commonly, the bacteria cause fever, malaise, weight loss and lymphadenopathy. Often, cats show no clinical signs. Dogs can be exposed via tick or flea vector and show signs similar to humans, including possible endocarditis. With the dog and cat being from a southern state, both animals were at risk of exposure from Bartonella species.

44
Q

A 4-year old male castrated Basenji presents for polyuria, polydipsia, and weight loss. Blood work shows P=2.5 (2.9-5.3 mg/dl), K= 3.1 (3.9-5.1 mEq/L), total CO2= 12 (17-25 mmol/L). The remainder of the blood work is within normal limits. Urinalysis shows 3+ glucose. Which of the following is your most likely differential diagnosis?

a. pyo
b. DM
c. pyelonephritis
d. Fanconi syndrome

A

The correct answer is Fanconi syndrome. Fanconi syndrome is an inherited disease in Basenjis. The disease involves renal tubular defects causing an abnormal loss of electrolytes and solutes leading to hypophosphatemia, hypokalemia, and metabolic acidosis. DM is less likely because serum glucose is normal. The lab abnormalities present in this dog are not consistent with pyelonephritis. Pyometra is not a viable choice as the signalment describes a male.

45
Q

A 7-year old male castrated cat presents to you for difficulty eating. On examination, you see that there are dental lesions on the buccal surfaces of several premolar and first molar teeth. The cat shows signs of discomfort when you palpate around these teeth and the surrounding gingiva appears inflamed. You suspect that the cat has odontoclastic resorptive lesions. You perform dental radiographs which show evidence of endodontic necrosis. Which of the following is the most appropriate treatment recommendation?

A

As described in this case, feline odontoclastic resorptive lesions commonly affects cats with increasing incidence as cats age. One or more lesions are found in about 50% of the domestic cat population over 5 years old. Lesions are often seen at the buccal surfaces of premolars and the first molar teeth at the gingival margin. Canine teeth can also be affected but usually lesions occur in the roots and the crown may appear normal.

There have been many theories about the cause of these lesions including a relationship to plaque-induced inflammation, microfractures of the cemental surface, and mineral deficiencies but most of these theories are no longer supported. It is now believed that abnormal formation or mineralization of cementum results in cemental resorption. There may be a relationship to high levels of vitamin D.

For treatment, there are reports of using alendronate, a bisphosphonate which inhibits demineralization of bone. Also, laser therapy has been used. However, neither of these treatments are currently accepted as standard care of this disease and extraction is the only current treatment that offers permanent prevention of pain to the patient. Restorative dental procedures are retained without recurrence in <25% of cats in 2-3 years.

In cases where lesions are entirely confined to the crown with no deep periodontal pockets and no radiographic evidence of endodontal necrosis, one can consider retaining the root. However, in the case described, complete extraction is the most appropriate recommendation. Because the incidence of lesions increases with age, it is likely that the cat in this case will develop lesions in other teeth in the future. Excessive Vitamin D should be avoided but other recommendations for prevention are controversial. Diligent dental care is certainly recommended.

46
Q

When performing an enterotomy in a cat, it is best to cut ________.

a. lat to antimesenteric border
b. on mesenteric border
c. On antimesenteric border
d. lateral to mesenteric border

A

C. On the antimesenteric border.

47
Q

You just began working in the intensive care unit and you notice a Cornish Rex exhibiting fever, restlessness, vomiting, urticaria, and hemoglobinuria. What do you think is happening?

a. blood transfusion rxn
b. diabetic ketoacidosis
c. disseminated IV coagulation
d. cyclosporine reaction

A

A. The correct answer is blood transfusion reaction. This is the perfect random question. The fun fact to remember is that Cornish rex, British short hair, and Devon rex cats are predisposed to having Type B blood type. Type B cats have anti A antibodies and tend to have acute transfusion reactions as compared with type A cats. Also, realize that the clinical signs being exhibited are those of anaphylaxis and that will help you narrow down the choices.

48
Q

A 13-year-old female spayed domestic short hair cat presents for a geriatric screening. The owner reports that the cat is polyuric, polydipsic, and slightly lethargic recently. Physical exam reveals a nodule in the region of the thyroid gland and a thin body condition. Blood work is run with the following findings (normal ranges are within parentheses): T4 2.7 mg/dL (2.3-4.7 ug/dL), blood urea nitrogen 34 mg/dL (10-32 mg/dL), creatinine 2.1 mg/dL (0.5-2.2 mg/dL), total calcium 16 mg/dL (8-11 mg/dL), phosphorus 2.5 mg/dL (3.5-8.1 mg/dL). An ionized calcium is 1.8 mmol/L (1.12-1.32 mmol/L). A urinalysis shows a urine specific gravity of 1.010 and numerous calcium oxalate crystals. An ultrasound of the neck confirms a nodule within the region of the thyroid. What do you tell the owner?

a. lymphoma, stage
b. hyper PTH, sx
c. hyperT, methimazole
d. renal failure, fluids
e. UTI, abx

A

B. The symptoms, physical exam findings, and blood work results are most consistent with primary hyperparathyroidism, which is most commonly caused by a parathyroid gland tumor. The elevated calcium causes the signs of PU/PD. Cats will commonly not show significant clinical signs and may be diagnosed incidentally on routine blood work evaluation.

This cat’s clinical signs of PU/PD and slight lethargy are non-specific, but classic for primary hyperparathyroidism along with the blood work shown in the question, which depicts borderline or mild azotemia, high total and ionized calcium levels, and a low phosphorus.

49
Q

Which insulin is most similar in composition to feline insulin?

A

Feline insulin is closest in its amino acid sequence to bovine insulin, differing by only one amino acid in the A-chain. Canine insulin however is identical to porcine insulin in its amino acid sequence. It is quite dissimilar from feline insulin, differing by four amino acids. Feline and human insulin sequences differ by four amino acids.

Although feline insulin is closest in sequence to bovine insulin, some cats can be managed quite well on other forms of insulin. Insulin glargine (Lantus), an engineered human insulin, is the current recommended insulin for cats. ProZinc (a protamine zinc recombinant human insulin) is also an option and has been FDA approved for use in diabetic cats. PZI (bovine zinc insulin) is no longer manufactured.

50
Q

An owner calls concerned that her 2 month-old kitten vomited a worm. You just saw the kitten for an exam the week before and gave a dose of Pyrantel during the appointment. According to the owner, the worm looks like a piece of spaghetti. What course of action do you take for the most likely diagnosis?

A

You would recommend to examine the worm and perform a fecal flotation for definitive diagnosis of the worm and to diagnose concurrent parasites. Most likely, based on the history and description, this patient is dealing with a Toxocara infection. Toxocara cati can be transmitted from queen to kitten via transmammary infection. Environmental control is also extremely important as grooming is a common source of egg ingestion. Due to the life cycle of roundworms, it is important to continue to treat every 2-3 weeks until the patient is 3-4 months old. It is unlikely that a one-time treatment of Pyrantel will resolve the issue.

51
Q

This 8-week old domestic short hair cat presented with an acute onset of severe chemosis as seen in the photo. The cat is systemically healthy otherwise with no oral or corneal lesions. Which of the following diseases most commonly causes severe chemosis in the absence of other systemic signs?

A

The correct answer is Chlamydophila felis. The key is to note the severe chemosis which is characteristic of chlamydophila infections. This upper respiratory tract pathogen does not cause oral ulcers in cats. Herpes virus will cause ocular, dendritic ulcers, and less commonly cause oral ulcers, and is commonly associated with systemic disease. Calicivirus also causes more systemic disease and can cause oral ulcers. Mycoplasma is not associated with such severe chemosis.

52
Q

An 8-month old male neutered domestic shorthair presents for ptyalism and decreased appetite. His temperature is normal. He has received his first set of vaccines from the Humane Society prior to adoption by this owner. Humane Society records indicate he was treated recently for upper respiratory symptoms. Upon oral examination you note severe gingivitis and stomatitis and multiple lingual ulcerations. FeLV and FIV testing were negative prior to adoption. Which of the following causes is suspected and which is the best treatment option listed?

A

Calicivirus is a common virus that can cause upper respiratory symptoms as well as oral ulcerations and stomatitis in cats. The stomatitis is often associated with concurrent bacterial infection of the mouth. An antibiotic such as clindamycin, doxycycline liquid, or amoxicillin-clavulanic acid (Clavamox) should be administered. Sucralfate in a slurry can help to coat the ulcerations for quicker healing. An analgesic, such as buprenorphine, should also be considered. An esophagostomy tube may be useful in severe cases when the cat will not eat on his own despite initial therapy.

A recent study found that 88% of cats with stomatitis were shedding both feline calicivirus and feline herpes virus-1, making these two viruses highly suspicious in playing a role in feline stomatitis.

Feline immunodeficiency virus (FIV) and feline leukemia virus (FeLV) do not appear to play as much of a role in this disease. Immunosuppression from these viruses make concurrent infection with calicivirus and herpesvirus more common.

A cause-effect relationship between Bartonella and feline stomatitis has not been proven but has been suspected in some cases. Doxycycline tablets should be avoided in cats due to risk of esophageal stricture.

There has been some controversy regarding the use of corticosteroids for stomatitis. Mostly, they have been used in cases of lymphocytic plasmacytic stomatitis that do not respond to other therapies and as a last resort. In general, the use of corticosteroids for stomatitis is not widely accepted.

53
Q

Treatment for giardia

A

fenbendazole or metronidazole

54
Q

Treatment for Polycystic kidneys

A

The image shows polycystic kidneys. This is an inherited disease that most commonly affects Persian cats. Other breeds such as Himalayans and British shorthairs are occasionally affected as well. The responsible gene is autosomal dominant. The homozygous form is lethal in utero and all affected living cats are heterozygotes.

The disease is diagnosed most reliably by ultrasound and treatment is similar to that of other causes of chronic renal disease with fluids, a low protein diet, gastric protectants (such as famotidine), maropitant, and appetite stimulants.

Cyclophosphamide, doxorubicin, vincristine, and prednisone are treatments for lymphoma, which can also affect kidneys bilaterally, but generally the lesions would not be cystic in appearance such as those in the picture.

Typically, the cysts are diffuse throughout both kidneys so surgical excision is not an option.

55
Q

A 4-month old kitten arrives at your clinic with a 3-day history of anorexia, lethargy, vomiting, and diarrhea. On physical exam the cat is 5% dehydrated and has a temperature of 103.9F (39.9 C). What is your primary differential?

a. FB
b. FIP
c. FIV
d. Panleuk

A

The correct answer is feline panleukopenia virus. These clinical signs most closely correlate with panleukopenia. One must piece together the signalment, history, and clinical findings. Usually a cat with a foreign body will not have diarrhea and a fever. It is unlikely for a 4-month old kitten to have FIV due to maternal antibody protection. It is also unlikely for FIP to present in such a manner. With the wet form of FIP, you may see dyspnea due to pleural effusion and abdominal distention due to ascites. With the dry form of FIP it will depend on the organ that is affected. You may see a hepatopathy, splenomegaly, renal failure, etc.

56
Q

White cats or cats with areas of white fur on the face or ears are predisposed to developing ___________ from UV light. These lesions are usually ulcerative and appear around the nose, ears, or eyelids.

A

Squamous cell carcinoma

57
Q

Which of these chemotherapeutic drugs cause fatal pulmonary edema in cats?

a. vincristine
b. doxorubicin
c. carboplatin
d. 5-fluoruracil
e. cisplatin

A

The correct answer is cisplatin. The famed statement, “cis-plat splats cats” is quite appropriate. 5-fluorouracil is also contraindicated for use in cats, but it is neurotoxic. Carboplatin, vincristine and doxorubicin are all used in cats.

58
Q

Treatment of Pemphigus

a. triamcinolone
b. itraconazole
c. prednisone
d. chlorambucil

A

B. itraconazole
In cats, the most commonly affected area is the head. Specifically, the pinnae, nasal planum, chin, or periocular region are involved. A smaller percentage of cats will also have lesions on their feet. Approximately 90% of cats with pemphigus foliaceus will have lesions in multiple sites, with over 75% having the head or face involved. Over 90% of cases will also have bilaterally symmetrical lesions. Although the condition is pustular in nature, these are usually not observed on clinical presentation; instead, crusts are seen as evidence of ruptured pustules. The condition is thought to be immune-mediated in nature with potentially a genetic component. Drug induced pemphigus foliaceus has been reported with the use of itraconazole. Interestingly, there is a case report of ampicillin inducing pemphigus foliaceus in a cat as well. All the other medications have been used for the treatment of this condition. A large retrospective study of 57 cats performed in 2003 documented that patients were more likely to respond with triamcinolone, as compared to prednisone or chlorambucil.

59
Q

What is the most appropriate way to initially manage a cat with pyothorax from a cat bite wound from several weeks ago after thoracocentesis?

A

The correct answer is to place a thoracostomy tube, lavage three times daily with LRS, appropriate systemic antibiotic for 6 weeks. If the cause of the pyothorax is identifiable, such as a foxtail, attempts should be made to treat that cause. In this instance, the initial wound is probably no longer visible or treatable. Aggressive management of these patients is necessary. Placement of a thoracostomy tube with continuous suction or intermittent aspiration is necessary. Lavage 2-4 times per day for 1 hour with an isotonic fluid should be performed. There is no advantage to adding antibiotics to the lavage fluid, although heparin is beneficial. Systemic antibiotics based on culture and sensitivity should be administered for a minimum of 4-6 weeks. If there is no improvement in 3-4 days, surgery is indicated to explore for abscesses, foreign bodies, etc. Pyothorax is an accumulation of pus in the pleural space. The fluid is classified as an exudate and the cells usually consist of neutrophils. If it is a bacterial pyothorax such as that from a cat bite, the neutrophils will be degenerate. Fungi and agents such as Actinomyces and Nocardia often cause exudates of non-degenerate neutrophils and macrophages. Reactive mesothelial cells are often seen in the effusion as well.

60
Q

A 10-year-old female spayed DSH presents to you after the pet-sitter found her with the following wound (ulcerative wound on L abdomen and chest). She is strictly indoors and the pet-sitter notes that she has otherwise been normal. She is the only animal in the house. She was previously diagnosed with diabetes mellitus and has been receiving her insulin on a consistent schedule but she continues to be polyuric and polydipsic. Her haircoat is unkempt and her abdomen is slightly pendulous. A CBC was unremarkable. Chemistry panel showed hyperglycemia and her urine specific gravity was 1.040. An abdominal ultrasound was performed. Which ultrasonographic finding is consistent with the likely underlying disease process?

a. hypoechoic pancreas and hyperechoic mesentery
b. bilateral adrenal atrophy
c. normal to enlarged adrenal glands
d. hyperechoic liver and moderate hepatomegaly

A

The clinical signs, patient’s medical history, and lack of overt trauma is highly suggestive of skin fragility syndrome. Typically, these patients appear to have a normal looking coat from a distance but when palpated or manipulated one must be very careful, particularly when scruffing the patient.

Skin fragility syndrome in cats is commonly associated with poorly regulated diabetes mellitus due to hyperadrenocorticism. Cushing’s disease can cause insulin resistance. Clinical signs are similar to those seen in dogs (pendulous abdomen, polyuria, polydipsia, polyphagia, lethargy, and muscle wasting). The hyperadrenocorticism can be iatrogenic, adrenal dependent or pituitary dependent. In cats, approximately 80% have a pituitary tumor while 20% have an adrenal tumor. ACTH stimulation test is not recommended in cats due to poor sensitivity. The low dose dexamethasone suppression test is recommended to confirm hyperadrenocorticism. Urine cortisol-creatinine ratio can be used as a screening tool. The UCCR has a high sensitivity so a negative test makes Cushing’s highly unlikely.

Abdominal ultrasound can be used to support your diagnosis. With pituitary dependent Cushing’s (which constitutes 80% of cats), you would expect normal to hypertrophied/enlarged adrenal glands. The excessive ACTH secreted from the pituitary gland produces an excessive amount of cortisol and adrenal hypertrophy.

Bilateral adrenal atrophy can be seen with hypoadrenocorticism. A hypoechoic pancreas and hyperechoic mesentery is highly suggestive of pancreatitis which is not associated with fragile skin in cats. A hyperechoic liver and moderate hepatomegaly can be suggestive of hepatic disease such as hepatic lipidosis or lymphoma which would not result in fragile skin either.

61
Q

A 1-year old female stray domestic short hair cat was presented for pruritus. Your physical exam revealed a generalized miliary dermatitis and dark brown flecks in the fur. Which of the following findings are most consistent with the most likely diagnosis?

a. yeast on impression smear
b. anemia, eos, tapeworm
c. mites on ear swab, eos, anemia
d. mites on skin scrape, eos, enemia

A

B.
The correct answer is anemia, peripheral eosinophilia, tapeworm infestation. The cat has flea allergy dermatitis, which often manifests as a miliary dermatitis. Anemia occurs when there is a large burden of fleas feeding on the cat, peripheral eosinophilia is seen with allergies or parasite infestations, and tapeworms are transmitted by the ingestion of fleas carrying tapeworm eggs. The brown flecks in the fur found on physical exam are flea dirt, which are the feces of fleas after ingesting blood. (We know this a somewhat vague question, but we feel you need to get accustomed to these types of questions.

62
Q

You see an 8-month old kitten with the effusive form of feline infectious peritonitis and perform euthanasia. The kitten was having severe diarrhea around the house when it became ill. The owner has a 2 year old cat at home and wants to know what this cat’s prognosis is since it has been exposed to the sick kitten. Currently this cat is clinically healthy. What do you tell her?

a. rec coronavirus titer
b. PCR feces to see if virus is shedding
c. highly contagious, may show symptoms w/in 2wk
d. not contagious
e. start L-lysine to suppress infection

A

D. Feline infectious peritonitis is not a contagious disease. It is a disease that is caused by a mutation of feline enteric coronavirus. It is unknown why in some patients this virus mutates and causes the FIP syndrome. It is most likely to occur in young or immunocompromised cats. Her other cat is not necessarily going to get FIP just from exposure. In fact, the majority of the cat population has been exposed to the feline enteric coronavirus.

Because most cats in the general population have been exposed, it makes interpretation of coronavirus titers difficult. The titers can be elevated due to prior exposure and not from FIP. The titers can only be interpreted in lieu of clinical signs, blood results, etc.
L-lysine is an anti-viral medication that may have some benefit for suppression of herpes virus but would not be a prevention or treatment choice for coronavirus.

The coronavirus is shed in the cat’s feces during active infection with coronavirus. Some infected cats do not shed the virus. The virus attacks the intestinal tract and causes GI upset. PCR on the feces would detect coronavirus, but does not distinguish between the enteric coronavirus and the mutated FIP form of the virus.

63
Q

You perform thoracocentesis on a cat with pleural effusion. The fluid has a hazy gold color and a protein content of 7 g/dl and 7,000 cells/uL. These cells are primarily non-degenerative neutrophils with lesser numbers of other white blood cells and mesothelial cells. What is the most appropriate interpretation of these findings?

a. chylous
b. neoplasia
c. FIP
d. exudate, pyothorax

A

c. The correct answer is that this effusion is strongly suggestive of FIP. FIP causes a classic straw or gold colored effusion with high protein content (about equal to serum protein) and moderate cell counts. An exudate would tend to have somewhat higher cell counts and lower protein. The protein content is not consistent with chyle or neoplasia.

64
Q

A woman brings her two cats into your clinic. She is worried because she has a young daughter who loves to play with her cats but she is concerned about Bartonella transmission resulting in cat scratch disease. You perform culture, PCR, and Western Blot on her cats, and those tests are negative. Which of these measures would be the most important way to decrease the chance of her cats acquiring Bartonella from another cat?

a. spay (std)
b. heartworm meds
c. prevent bites/scratches from other cats
d. strict flea control
e. frequently clean litterbox

A

d. While transmission of Bartonella from cats to humans is primarily via scratches and occasionally bites, the same is not true for cat-to-cat transmission. Arthropod vectors are the main mode of transmission, particularly the cat flea, Ctenocephalides felis. Direct transmission from cat-to-cat or vertically from queen to kitten in a flea-free environment is very unlikely.

65
Q

Treatment for hypertrophic cardiomyopathy in cats (as depicted in the pathologic image) is aimed at which of the following?

improve systolic v. diastolic function?
via inc. or dec. HR?

A

The correct answer is improving diastolic filling by decreasing heart rate. The primary problem with HCM is thickening of the wall of the left ventricle, impairing diastolic filling. There is usually no problem with contractility and systolic function. Slowing the heart rate provides for longer diastolic filling, allowing the left ventricle to fill more effectively. Decreasing the heart rate also decreases the severity of systolic anterior motion of the mitral valve.

For this reason, the medications used most frequently to slow the heart rate are:

1) Beta- blockers (i.e. atenolol: 6.25-12.5 mg/cat orally qD-BID, or metoprolol: 0.5-1 mg/kg TID)
2) Calcium channel blockers: diltiazem: 1-2 mg/kg TID (PO) or 7.5 mg per cat TID

For cases with severe left atrial enlargement or cats in heart failure, also consider:

3) ACE inhibitors (i.e. enalapril or benazepril: 0.25-0.5 mg/kg once daily)
4) Diuretics: furosemide (Lasix): 1-2 mg/kg BID-TID
5) Preventing thromboembolic disease with aspirin or clopidogrel (Plavix).

Also, always keep in mind that you want to identify and treat possible underlying hypertension or hyperthyroidism that could be contributing to the disease.

66
Q

A 2-year old male neutered cat presents to you depressed, hypersalivating, and ataxic with muscle tremors. The owner reports that a pyrethrin-based spot-on formulation for flea control belonging to their Golden Retriever was accidentally applied on the cat earlier today. Which of the following drugs will you use to treat the cat’s clinical signs?

a. atropine
b. 2-pam
c. ace
d. amoxicillin
e. methocarbamol

A

Pyrethrins alter the activity of the sodium ion channels of nerves, which prolongs the period of sodium conductance. This increases the length of depolarization resulting in repetitive nerve firing. Cats are particularly sensitive to pyrethrin-containing products and can develop clinical signs within hours after administration. Affected animals should be bathed to remove remaining product. Minor clinical signs such as hypersalivation and ear twitching are usually self-limiting and do not require treatment. Control of marked tremors or seizures can be achieved with methocarbamol (Robaxin).

67
Q

A 12-year old male neutered domestic short hair cat presents for ongoing evaluation of diabetes mellitus. The cat was diagnosed 6 months ago and has continued to be markedly polyuric, polydipsic, polyphagic, and has been gaining weight. The cat is currently receiving 10 units of glargine insulin every 12 hours. On physical exam, the cat weighs 15 pounds (6.8 kg) and has an enlarged head, abdomen, and paws. What imaging modality would be most appropriate to try and prove what you suspect is causing the uncontrolled diabetes and weight gain in this cat?

A

This cat has the signs and symptoms of acromegaly. Acromegaly is caused by excessive growth hormone release from the pars distalis from a tumor in the pituitary gland. Excessive growth hormone causes a defect in the insulin receptors on target cells causing insulin resistant diabetes mellitus. The enlarged head, paws, abdomen, and weight gain despite uncontrolled diabetes is due to the anabolic effects of the growth hormone.

Treatment for this condition includes radiation therapy to the pituitary tumor, high doses of insulin to try and control the diabetes, and somatostatin analogs (octreotide) to try and inhibit the release of growth hormone from the tumor. Surgical excision has been used as a form of treatment in people with pituitary tumors, but this has only been rarely reported in cats.

68
Q

Which of the following is NOT a common post-operative complication of thyroidectomy for hyperthyroidism in cats?

a. lar par
b. esophageal stricture
c. hypocalcemia
d. horners syndrome
e. hypothyroidism

A

The correct answer is esophageal stricture. Esophageal stricture is not a common post-op complication of thyroidectomy. Hypocalcemia can occur due to damage or excision of the parathyroid glands. Horner’s syndrome occurs when the sympathetic trunk running through the neck is damaged. Laryngeal paralysis occurs with damage to the recurrent laryngeal nerve running through the neck. Hypothyroidism can occur secondary to removal of the affected thyroid gland or glands.

69
Q

Mousie, a 3-year old FS DLH, has been vomiting once daily for the past week. Clinically, she otherwise acts normally. Bloodwork and radiographs are within normal limits. The owners have seen chunks of hair in her vomit. She is currently on no medications and is eating Fancy Feast. Which of the following would you recommend?

a. metoclopramide
b. lactulose
c. pred
d. laxatone
e. maropitant

A

This young long-haired cat is apparently having a hairball problem. Frequent brushing, a higher quality diet, and a hairball treatment such as Laxatone (petroleum/mineral oil gel) should be recommended for this cat.

Lactulose is a stool softener often used to help with constipation.

Metoclopramide is an anti-emetic and promotility agent and may be beneficial for hairballs if a high quality diet and laxatone fail to correct the problem.

Maropitant (Cerenia) is a central and peripheral acting anti-emetic approved for the use in dogs.

Prednisolone is a corticosteroid which could be used for treatment of underlying inflammatory conditions.

70
Q

Krissy, a 10-year old female spayed Himalayan, has had chronic constipation problems over the last year and has been diagnosed with megacolon. Today she presents for vomiting and straining to defecate. She is currently taking Lactulose to help with her bowel movements. You palpate hard feces in the colon. You give her an enema and manually evacuate most of the hard stools. Which of the following medications could also be used to help treat and try to prevent this problem in the future?

a. metronidazole
b. cisapride
c. omeprazole
d. sulfasalazine
e. prednisolone

A

b. There are five main treatment objectives for cats with megacolon. These include adequate hydration status, removal of impacted feces, laxative therapy, promotility agents for the colon, and dietary fiber.

This cat may benefit from Cisapride, which is a benzamide prokinetic drug. Due to the chronic stretching of the colon from the feces impactions, the colon can no longer move the feces out of the body in a normal way. This medication has anecdotally been shown to help cats evacuate feces more efficiently, especially in mild or moderate cases. Some cats may also benefit from a fiber source such as psyllium or canned pumpkin.

If the constipation problems continue despite the above mentioned treatments, colectomy should be considered.

Prednisolone is used sometimes in treating inflammatory bowel disease, but for megacolon is not indicated. Metronidazole and Sulfasalazine are antibiotics sometimes used for diarrhea or inflammatory bowel disease.

Omeprazole is a proton pump inhibitor used for prevention of gastric ulcers and upper GI disease and would not be helpful for megacolon.

71
Q

Which of the following is caused by taurine deficiency in cats?

a. iris atrophy
b. retinal degeneration
c. anterior uveitis
d. glaucoma
e. cataracts

A

The correct answer is retinal degeneration. Taurine deficiency in cats causes retinal degeneration as well as dilated cardiomyopathy. Since most commercial cat diets have adequate taurine supplementation, it is now rare to see this occurrence unless the cat is on a home-cooked diet.

72
Q

A 6-month old domestic short hair cat presents for its first physical exam. Cardiac auscultation reveals a grade IV/VI holosystolic murmur on the right thorax. Thoracic radiographs were unremarkable and echocardiography showed a small turbulent jet flowing through the ventricular septum from the left ventricle to the right ventricle. Which of the following do you tell the owner?

A

The correct answer is the prognosis is very good with small ventricular septal defects and no treatment is needed. The prognosis of small VSDs is very good. Some will close by themselves within the first year of life. It is unlikely that a small VSD will cause any significant problems such as heart failure, though follow-up evaluation is warranted to monitor cardiac size and function. Large VSDs carry a guarded prognosis and require cardiac bypass for surgical correction or novel device closure via transcatheter techniques; neither of which are commonly employed in animals. Small VSDs cause a relatively more turbulent jet of blood through the small defect, causing a louder murmur than a large VSD.

73
Q

Rubiosis iridis as seen in this cat is a sign of what process in the eye?

a. Ant uveitis
b. glaucoma
c. lens luxation
d. iris atrophy
e. Chorioretinitis

A

a. Anterior uveitis
Along with other signs such as aqueous flare, hyphema, hypopyon, keratic precipitates, dec. intraocular pressure.
*Caused by infection, immune-mediated, traumatic, idiopathic

74
Q

A 1-year-old male neutered cat is transferred to you in the morning after seeing an emergency service for electrical cord bite trauma. You anesthetize the cat to address numerous oral burns and ulcers and note frothy pink fluid accumulating in the endotracheal tube (see image). What is the likely cause of this?

a. tracheal tear
b. congestive heart failure secondary to electric cord bite
c. pleural effusion
d. tube in esophagus
e. non-cardiogenic pulmonary edema

A

E. non-cardio pulmonary edema
Electrical cord trauma commonly causes burns and ulcerations of the mouth, lips, and tongue. Due to the effect of electrical current on capillaries, non-cardiogenic pulmonary edema is an important complication to expect and monitor for. Although electrical cord bites can cause cardiac arrest, congestive heart failure would not be an expected complication.

75
Q

A 3-year old indoor/outdoor domestic short hair cat presents with a history of lethargy and decreased appetite. T= 103.1 F/ 39.5 C, P= 200 bpm, R= 36. On exam you note a swelling over the bridge of the nose and on fundic exam see some dark circular lesions in the retinas. The disease you suspect in this cat may have been transmitted by which of the following?

a. inhalation of spores from pigeon droppings
b. ingestion of oocysts from raw meat
c. puncture wound from contaminated soil
d. scratch from another cat

A

Cryptococcus is a genus of encapsulated yeast that is often associated with or found in pigeon droppings and eucalyptus trees. The spores are most often inhaled from the environment where these are present. Cats with cryptococcus will often develop a swelling over the bridge of the nose and lesions in the retinas.

Sporothrix is a fungus found in the soil (“rose grower’s disease”) and is introduced into the body via a puncture from a thorn or through an open wound or cut when exposed to contaminated soil.

Toxoplasmosis may be transmitted through ingestion of raw meat.

76
Q

A 1-year-old female indoor cat from Florida was presented for an after-hours emergency examination. Approximately, 72 hours after spending the night locked out on the family’s screened-in porch, the cat developed acute severe facial pruritus and was presented for the lesions (ulcerated nasal planum). Similar lesions were seen on the ear tips and paws; only thinly haired areas with dark hair were affected. The cat was otherwise healthy. Skin scrapings were negative. Impression smears of the nose revealed inflammatory exudates comprised of approximately 75% eosinophils and lesser numbers of neutrophils, lymphocytes, and mast cells.

Which of the following is the best diagnostic or treatment plan?

a. aggressive flea control
b. dietary trial with novel protein source
c. confine indoors for 5-7 days
d. intradermal skin testing
e. bacterial culture of lesions

A

c. confine indoor.
The history, onset, clinical appearance, as well as the cytologic and histologic findings are most consistent with insect bite hypersensitivity. There are other reasonable but less likely differentials including pemphigus foliaceus, food allergy or atopy, and dermatophytosis.

The best way to rule in/out the most likely diagnosis is to confine the cat indoors and see if the lesions resolve as none of the other differentials would be expected to respond. It would be premature to institute skin testing for allergies or a dietary trial for food allergy. Aggressive flea control is not a bad idea, but the lesion distribution is less likely for flea allergy dermatitis. Cats are very sensitive to several pesticides. Bacterial culture would likely result in growth of normal superficial bacteria and not indicate the underlying pathology.

It also may be necessary to initiate an anti-pruritic therapy to reduce further self-trauma by the cat.

77
Q

You are called out on a house-call to see the 3-year-old male Labrador Retriever shown in the photo below. He presents for bilateral periocular edema (see photo) and facial pruritus. This has been a recurrent problem for the last 4 months. The owners reported the dog would be normal for several days, and then acutely develop the periocular swelling and itching. These episodes always happened while the dog was inside the house.
The previous episodes would last for several hours, and then the edema would resolve en route to the veterinary clinic. After each episode, the dog had some residual periocular or facial pruritus for 1-2 days.

You perform conjunctival swabs and aspirates of the edematous area which reveal large numbers of eosinophils.

What is the most likely underlying cause of the dogs signs?

a. bacterial infection
b. Neoplasia
c. autoimmune
d. parasitism
e. hypersensitivity

A

Edema as you see here is a common clinical sign of a type I hypersensitivity reaction or allergy. The intermittent nature of the episodes suggests some type of allergic reaction, most likely infrequent contact with a potent allergen to which the dog has become sensitized. An underlying allergic etiology is further supported by the presence of eosinophils.

Identifying the underlying allergy in cases like this one can be challenging, and a thorough history and investigation of the dog’s environment is warranted.

The other answer choices presented are less consistent with the recurrent and sudden clinical signs that are repeatedly observed.

78
Q

A 4-month old intact male Collie presents for severe dermatitis along his nose and ears (see image). The areas are sensitive on palpation but not pruritic. No other areas are affected. Skin scraping and cytology are negative for mites or bacteria. Fungal culture is negative. Anti-nuclear antibody tests are ordered and are also negative. How do you educate the owner based on your presumptive diagnosis?

a. familial, cyclic, will regress
b. not curable, discuss euthanasia
c. biopsy and culture, long term tx needed
d. long term immunosuppressives

A

Familial dermatomyositis is a familial disease seen in Collies, Shetland sheepdogs, and other collie breeds. Diagnosis can be made with skin or muscle biopsies, but can be difficult as the clinical and histopathologic signs can wax and wane. Inciting causes can include vaccinations, sunlight, viral infections, or drugs. Some cases; however, will come and go without obvious inciting causes. Some cases can become severe enough to include muscle atrophy; megaesophagus has also been seen with this disease.

Treatments are often symptomatic and supportive and include Vitamin E supplements, Omega 3 fish oils, treatments for any secondary bacterial infections, and pentoxifylline. Prednisone can also be used to get more severe cases under control. Avoiding prolonged sun exposure can also help.

Educating the owner on the fact that signs will reoccur despite treatment can alleviate some owner frustrations. Often times signs will regress with or without treatments. Educate owners on neutering animals to prevent propagation of diseased animals.

79
Q

tx cat hcm?

a. nitroglycerin paste
b. furosemide
c. atenolol
d. spironolactone
e. digoxin

A

This case describes typical findings for a cat with hypertrophic cardiomyopathy (HCM). Nitroglycerin, oxygen, spironolactone, and furosemide are indicated for cats with heart FAILURE from HCM, which this cat does not have.
Use of a beta-blocker such as atenolol is advocated by some cardiologists, particularly when left ventricular outflow obstruction exists. Atenolol alleviates the obstruction by reducing heart rate, LV contractility, and also reduces myocardial oxygen demands. Diltiazem and antithrombotics may also be considered.

Digoxin is used to slow atrioventricular conduction in the setting of supraventricular tachyarrhythmias or for systolic dysfunction. As the cat does not have a supraventricular tachyarrhythmia and has normal fractional shortening, digoxin is not indicated for this cat.

80
Q

A 10 year old female spayed Labrador retriever mix presents for regurgitation and weight loss. Thoracic radiographs show an 8 cm cranial mediastinal mass and a gas filled esophagus. What is the most likely diagnosis of the mediastinal mass?

a. Thymoma
b. Chemodectoma
c. Lymphoma
d. Thyroid carcinoma

A

a. The megaesophagus is caused by myasthenia gravis, which is occasionally seen with thymomas. Treatment of choice for thymomas is surgical excision. The myasthenia gravis typically resolves after removal of the thymoma. Although lymphoma is a reasonable differential for a mediastinal mass, the concurrent megaesophagus is more consistent with the scenario above.

Chemodectomas are heart base masses. An ectopic thyroid carcinoma could occur in the mediastinum but is rare and would be unlikely associated with megaesophagus.

81
Q

EPI in cats v. dogs

A

The correct answer is pancreatic acinar atrophy. This is an idiopathic condition where the exocrine cells of the pancreas atrophy with minimal inflammation. Chronic pancreatitis is a much more common cause of EPI in cats than dogs. For this reason, dogs with EPI do not usually have concurrent diabetes mellitus because the endocrine cells are spared, whereas cats often do have concurrent DM. Neoplasia and infection are rare causes of EPI.

82
Q

Brodifocum toxicity

A

Vitamin K Antagonist
CS: hemorrhaging
Tx: induce emesis (confirmation if green dye seen in vomitus or diarrhea), activated charcoal, rinse conjunctiva
Vitamin K1 supplementation for 4-6 weeks

83
Q

A man presents his middle aged female spayed indoor/outdoor cat for having difficulty breathing. Her gums are pink but she has moderate tachypnea and temperature is 104.1 F (40.1 C). The lung sounds are quiet. You perform a thoracocentesis and retrieve a pleural effusion that is white with a yellow tinge. It has a foul odor when you empty the syringe. What treatment do you advise?

A. Drain the effusion via thoracocentesis and start furosemide for congestive heart failure
B. Place a chest tube to drain and lavage chest and administer IV antibiotics
C. Culture the fluid and start antibiotics, the fluid will resorb after the infection is treated
D. Euthanasia due to the cat likely having FIP and the prognosis is grave
E. Drain the fluid via thoracocentesis, start a low fat diet and benzopyrone (Rutin)

A

This cat has pyothorax which is an accumulation of pus within the pleural cavity. In cats, the most common cause is a bite wound that introduces bacteria into the chest cavity. It can also be seen with migrating foreign body, or extension of pneumonia into the pleural space. At minimum, hospitalization with chest tube, drainage and lavage of the pleural space, and antibiotics are needed. Sometimes surgical exploration is required.

While this cat could have FIP, the foul odor and presence of bacteria indicate a suppurative infection which is not typical for FIP.

Congestive heart failure can present as pleural effusion in cats, but fever and this type of effusion are not typical of heart failure.

Culturing the fluid is important. However, the fluid will not resorb and the fluid must be removed and the pleural space lavaged.

A low fat diet and Rutin are treatments for pleural effusion caused by chylothorax which is an accumulation of lymphatic fluid within the pleural space.

84
Q

Which of these fungal pathogens grow in pigeon feces and has the potential of being a serious zoonotic problem?

 Cryptococcus 
 Aspergillus 
 Coccidioides
 Histoplasma
 Sporothrix
A

The correct answer is Cryptococcus. Cryptococcus neoformans grows exceptionally well in pigeon feces. Blastomyces is also found in bird feces. Histoplasma can grow in bird feces but is not as much of a zoonotic concern.

85
Q

A 10-year old feline domestic short hair presents to you in respiratory distress. A chest radiograph reveals rounded lung lobes and a significant bilateral pleural effusion. A sample of this fluid is milky white with an elevated triglyceride:cholesterol ratio. What is your immediate treatment choice for this cat?

a. Tap both sides of the chest to evacuate as much fluid as possible
b. Discuss the poor prognosis with the owners and offer euthanasia
c. Tap one side of the chest only to reduce likelihood of complications with tapping
d. Hospitalize this cat and place it in an oxygenated cage for medical treatment

A

a. The radiographs describe significantly rounded lung lobes which suggests that the fluid has caused some fibrosing pleuritis. Sheets of fibrous connective tissue often times will cause pocketing of fluid on the right and left sides of the chest. Thus, it is important to tap both sides of the chest to adequately drain enough fluid to relieve dyspnea. The immediate life-saving procedure will be to remove the fluid that is causing the respiratory distress. Chylothorax is usually idiopathic in origin and rarely may resolve spontaneously in 1-2 months. Most cases ultimately require surgery which is usually a combination of ligation of the thoracic duct and a sub-total pericardiectomy.

86
Q

In which of these conditions would you expect to find hypocalcemia in a cat?

T-cell lymphoma
Primary hyperparathyroidism
Hypervitaminosis D
Eclampsia

A

d. The correct answer is eclampsia. This occurs when serum calcium drops during parturition or lactation such that these functions cannot be supported. The other answer choices are causes of hypercalcemia.

87
Q

What is the treatment of choice for traumatic proptosis of the eye as shown in this image?

a. Tarsorrhaphy
b. Conjunctival flap
c. Push globe back into orbit with sterile blunt instrument
d. Phacoemulsification
e. Exenteration

A

a. The answer is tarsorrhaphy. Tarsorrhaphy is the treatment of choice for proptosis. In this procedure, the eyelids are pulled in front of the proptosed globe rather than pushing the eye in and then temporarily suturing the lids shut. Exenteration is a procedure used to remove eyes with neoplasia that is infiltrating into the surrounding structures. A conjunctival flap is a procedure usually used to help healing of complicated corneal ulcers. Phacoemulsification is the procedure used to remove cataracts.

88
Q

Rubiosis iridis as seen in this cat is a sign of what process in the eye?

 Iris atrophy
 Chorioretinitis 
 Lens luxation
 Glaucoma
 Anterior uveitis
A

The correct answer is anterior uveitis. Rubiosis iridis along with other signs such as aqueous flare, hyphema, hypopyon, keratic precipitates, and decreased intraocular pressure are all suggestive of anterior uveitis which can be caused by a number of infectious, immune-mediated, traumatic, and idiopathic causes.

89
Q

Which of the following is true about canine hypothyroidism?

a. It is often congenital
b. It is often caused by immune mediated lymphocytic thyroiditis
c. It can be caused by deficiency of thyrotropin-releasing hormone (TRH) by the hypothalamus
d. It is rarely caused by idiopathic atrophy of the thyroid parenchyma

A

b. About half of all cases of primary canine hypothyroidism are caused by immune mediated lymphocytic thyroiditis. Antithyroglobulin antibodies are present in about 35-50% of hypothyroid dogs. Idiopathic atrophy of the thyroid follicles is another common cause of the disease. Deficiency of TRH (tertiary hypothyroidism) from the hypothalamus has not been documented in dogs. Congenital hypothyroidism (cretinism) is rare in dogs.

90
Q

You are examining the eyes of a dog with all the signs of Horner’s syndrome. After instilling topical hydroxyamphetamine, there is no change. You then instill topical phenylephrine and in 5 minutes, the eye returns to normal. What can you conclude about this dog’s Horner’s?

a. He has a pre-ganglionic lesion
b. The Horner’s is idiopathic in origin
c. The Horner’s has only been present for less than 24 hours
d. He has a post-ganglionic lesion Correct Answer
e. The Horner’s has been present for a long time

A

d. The answer is that he has a post-ganglionic lesion. Hydroxyamphetamine acts by encouraging release of norepinephrine from the neuromuscular junction. If there is a pre-ganglionic lesion, NE will still be released by the postsynaptic neuron and signs will resolve. If there is a post-ganglionic lesion, there will be no NE to release. Also, with a post-ganglionic lesion, denervation hypersensitivity to NE occurs. As a result, adding phenylephrine topically can cause a rapid response. With pre-ganglionic lesions, the response usually takes greater than 20 minutes.

91
Q

Clinical signs of Iris atrophy

A
  • It leads to scalloping of the iris margin and sometimes a moth-eaten stroma.
  • absent or incomplete PLRs, dyscoria, or anisocoria.
92
Q

Which of the following is not a likely cause for hemorrhagic pericardial effusion in the dog?

a. Hemangiosarcoma
b. Congestive heart failure
c. Chemodectoma
d. Idiopathic hemorrhagic effusion
e. Left atrial rupture due to chronic mitral regurgitation

A

The correct answer is congestive heart failure. CHF usually results in a modified transudate pericardial effusion. Idiopathic hemorrhagic effusion and neoplasia are the most common causes for hemopericardium. Rupture of the left atrium due to chronic mitral regurgitation or trauma is another albeit more rare, cause.

93
Q

The 2-year-old Doberman Pinscher shown in the photo presents for hair loss of unknown etiology. Previous diagnostic tests were normal or negative and included skin scrapings, dermatophyte culture, complete blood count, urinalysis, serum chemistry panel, thyroid hormone evaluation, low-dose dexamethasone suppression test, and surgical neutering. Dermatological examination reveals a thin hair coat, nodular-like hair follicles, comedones, bacterial pyoderma, and scaling (see image). All of the other littermates are similarly affected. What is the most likely primary condition

a. Pemphigus vulgaris
b. Hypothyroidism
c. Color dilution alopecia
d. Hyperadrenocorticism
e. Systemic lupus erythematosis

A

Color dilution alopecia is a genetic disorder of the hair coat commonly associated with a blue or fawn coat color as is the case for the blue Doberman shown in the photo. The cause is unknown, but it is believed to be due, in part, to a defect in the coat color genes. Affected dogs are born with normal hair coats and, as they mature, they develop hair loss on the dorsum, recurrent bacterial infections, generalized thinning of the hair coat, secondary seborrhea, and cystic or dilated hair follicles.

The keys to answering this question are to recognize the color of the Doberman in the photo and concurrent alopecia and skin lesions. The information that all of the littermates are affected should have given you further reason to consider a genetic cause. The endocrine conditions (hypothyroidism and hyperadrenocorticism) are less likely due to the dog’s young age and having multiple affected littermates. In addition, previous diagnostic screening tests for those conditions were reported to be negative. The lesions are not consistent with pemphigus vulgaris and systemic lupus.

There is no effective treatment for the genetic disorder and affected dogs will continue to lose hair. Many are alopecic by 2-3 years of age. Control of the secondary pyoderma is important. Diagnosis can be confirmed by histopathology which shows melanin clumping in the epidermal and follicular basal cells, macromelanosomes in the hair shafts or hair bulbs, and follicular dysplasia. The melanin clumping and macromelanosomes are key histologic findings for diagnosis of this disease.

94
Q

Which of these is not a proper treatment for glaucoma (primary or secondary) in a dog?

a. Oral methazolamide (carbonic anhydrase inhibitor)
b. Topical mannitol
c. Topical latanoprost (prostaglandin analog)
d. IV mannitol
e. Topical dorzolamide (carbonic anhydrase inhibitor)

A

The answer is topical mannitol. Carbonic anhydrase inhibitors act by decreasing aqueous production which is partially dependent on the conversion of carbon dioxide to bicarbonate. It can be used topically or systemically. Latanoprost (Xalatan) is a prostaglandin analog frequently used topically in the treatment of glaucoma. IV mannitol works to decrease IOP through its properties as an osmotic attractant. It is not used topically.

95
Q

A 4-year old male castrated cat presents to you with the primary complaint of lethargy. The cat’s mucous membranes appear pale on physical examination. You perform bloodwork and find a hematocrit of 18% (30-45%). The anemia is regenerative with 3% reticulocytes (0-0.6%). You are concerned about Mycoplasma infection; which of the following tests would be most useful in ruling in or out such an infection?

a. Coombs’ test
b. PCR for Mycoplasma spp
c. New methylene blue staining
d. Coagulation tests
e. Bone marrow cytology or biopsy

A

b. The PCR test for Mycoplasma can be used to detect circulating DNA from Mycoplasma haemofelis and Mycoplasma haemominutum (previously known as Hemobartonella felis). Therefore, it is useful for determining if there is an active infection. In some cases, a fresh blood smear may allow detection of the organisms, but they frequently detach from RBCs when blood is placed in EDTA anticoagulant.

Since the anemia is regenerative, it is unlikely that you will need to perform a bone marrow evaluation. New methylene blue staining for reticulocytes or Heinz bodies will not rule in/out Mycoplasma, although increased numbers of Heinz bodies would be associated with other causes of anemia.

96
Q

Which of these is not a sign of Horner’s syndrome in dogs?

3rd eyelid protrusion
Enophthalmos
Ptosis
Mydriasis

A

The answer is mydriasis. Horner’s syndrome is denervation of the sympathetic fibers going to the eye. As such, you observe the opposite of what you would see in a fight or flight response. The signs of Horner’s are miosis, ptosis, 3rd eyelid protrusion, and enophthalmos.

97
Q

Idiopathic Trigeminal Neuritis

A

A syndrome seen in dogs with peracute onset of a dropped jaw and inability to close the mouth.
● No other sensory deficits or other cranial nerve deficits are present.
● Dogs are typically normally alert and responsive without difficulty swallowing food
that is placed on the back of the tongue.
● Should be distinguished from these differentials (usually based on speed of onset and lack
of other signs):
a. Mandibular fracture
b. Rabies
c. Neoplasia of mandibular nerve
d. Masticatory muscle myositis
● Treated with supportive care with fluids and hand feeding of soft food a prognosis is excellent, with
most dogs regaining function within 1-2 weeks and returning to normal within 3-4 weeks

98
Q

Idiopathic facial n. paralysis

A
● Acute onset of several signs.
a. Inability to blink (lack of menace or palpebral response).
b. Drooping lip and ear.
c. Drooling from one side of the mouth.
d. Dry eye.
● Facial sensation is normal.
99
Q

The image of the eye below is from a canine patient diagnosed with keratoconjunctivitis sicca (KCS). This condition causes a decrease in tear production which then results in the classic changes observed in this image. Which of the following is not an etiology of the disease?

a. Immune mediated
b. Penicillin toxicity
c. Long-term atropine administration
d. Sulfonamide toxicity

A

b. In canine patients, the most common cause of KCS is immune mediated. This is why these patients are treated with cyclosporine, a t-cell modulator. Penicillins have not been documented to result in KCS. Sulfonamide containing antibiotics, such as TMS have been shown to cause KCS in some patients. Long-term atropine administration has also been shown to result in KCS. Atropine is a parasympatholytic and will result in decreased tear production. Etodolac has also been associated with KCS in dogs. The corneal pigmentation seen is a nonspecific response to chronic irritation or inflammation. There is also evidence of corneal vascularization which occurs as part of the inflammatory and healing response.

100
Q

When performing surgery on a patient with compromised liver function what is a test that should be performed prior to surgery?

 Bile acids
 Coagulation panel
 PIVKA
 White blood cell count
 Blood urea nitrogen
A

The correct answer is coagulation panel. If you have compromised liver function, the liver may not be able to produce enough coagulation factors; thus a surgical procedure would bring the risk that the animal could bleed to death.

101
Q

4 components of nephrotic syndrome?

A

The four components of nephrotic syndrome are proteinuria, hypoproteinemia, hypercholesterolemia, and ascites or edema. This syndrome occurs with protein-losing nephropathies such as glomerulonephritis or amyloidosis. Hypercoagulability is not a component of nephrotic syndrome, although it can occur with protein-losing nephropathies due to the loss of antithrombin III.

102
Q

A 7-year old female spayed Golden Retriever that you suspect has Addison’s disease has bloody diarrhea, inappetance, and is dehydrated. What should be your next step?

a. Start aggressive IV fluid therapy and run a chemistry panel
b. Give IV dexamethasone and subcutaneous fluids
c. Give Lysodren
d. Run a chemistry panel and perform an abdominal ultrasound.

A

a. The correct answer is start aggressive IV fluid therapy and run a chemistry panel.

Addisonian patients often present in hypovolemic shock, so the first step to treating this dog is to restore vascular volume with IV fluids and to run a chemistry panel to check the extent of electrolyte abnormalities (hyponatremia, hyperkalemia, and elevated BUN). IV dexamethasone would not take precedence over restoring vascular volume, and subcutaneous fluid treatment is not aggressive enough for treatment of hypovolemia. Abdominal ultrasound would not be a priority in this case but could eventually be helpful in ruling in/out other differentials.

Lysodren is used for treating hyperadrenocorticism. One possible complication of Lysodren treatment is to make a dog Addisonian, which often causes the animal to present with the signs described as in the question.

103
Q

You diagnose a deep pyoderma. She is started on a limited ingredient diet, chlorhexidine shampoo, and you need to send oral antibiotics. Which of the following treatments would be appropriate?

A

c. cephalexin for 8 weeks.
Deep pyodermas involve tissues deeper than the epidermis, including the dermis and even subcutis. The skin may heal on the surface before the deeper infection is resolved; making clinical cure difficult to assess. In general, deep infections can require 6-8 weeks of antibiotic treatment (and even 12 weeks for severe cases) for resolution.

Deep pyoderma can be secondary to allergies, skin fold anomalies, endocrine disorders, immune mediated skin diseases, bacterial and/or fungal skin infections, or migrating foreign bodies. Determining the underlying etiology will help with resolution and minimize chances of treatment failure and re-occurrence.

Diagnostics should include skin scraping and impressions, culturing for bacteria and fungi, biopsy, and blood work. Treatments typically include antimicrobials that have demonstrated effectiveness, antifungals if needed, and frequent topical treatments with chlorhexidine shampoo and removal of the dead tissue and debris.

Incidentally, the German Shepherds are predisposed to severe deep pyodermas that can be difficult to treat. Discussing this with your client will help give them realistic expectations for treatment. German Shepherds are at a higher risk of developing dry eye from Trimethoprim-sulfa; therefore, it is not recommended for this condition due to the length of time required to treat a deep pyoderma.

104
Q

Which of these is the most important treatment for salmon poisoning?

Praziquantel
Oxytetracycline
Fenbendazole
Ivermectin

A

The correct answer is oxytetracycline. Because the signs are caused by a rickettsial organism, Neorickettsia helminthoeca, treatment is with a tetracycline-type of drug. Praziquantel is usually given to eliminate the fluke even though it does not usually cause any clinical signs in order to prevent contamination of other waterways.

105
Q

Which of the following is true about diltiazem?

a. Diltiazem slows the heart by inhibiting the influx of calcium into the myocardial cells.
b. Diltiazem speeds up the heart because it is a beta adrenergic agonist.
c. Diltiazem slows the heart by antagonizing beta adrenergic receptors.
d. Diltiazem speeds up the heart rate by increasing the influx of calcium into the myocardial cells.

A

The correct answer is diltiazem slows the heart by inhibiting the influx of calcium into the myocardial cells. Diltiazem is a calcium channel blocker used to slow the heart to treat supraventricular tachycardias, hypertrophic cardiomyopathy, and hypertension.

106
Q

You ask your technician to give a 1 ml/kg dose of lidocaine to a dog that is having ventricular premature complexes. You realize after it is too late that she miscalculated the dose and gave 10 times what you asked for. What is the most common early sign of lidocaine toxicity in dogs?

 Peripheral neuropathy
 Profound bradycardia 
 Anaphylaxis
 Central nervous system depression 
 Apnea
A

In dogs, toxicity of lidocaine is manifest primarily as CNS signs. Drowsiness or agitation may progress to muscle twitching and convulsions at higher doses. This occurs before respiratory or cardiac depression. Hypotension may develop if an IV bolus is given too rapidly.

Cats are more sensitive to lidocaine toxicity and may show cardiac suppression and CNS excitation.

107
Q

An 8-month male intact English bulldog presents after his owner noted bleeding from the penis. He took a look and describes seeing a small mushroom-like mass at the tip of the penis. The owner states that the dog has always been healthy and that the only thing unusual about the situation is that the dog masturbates on his favorite stuffed animal on a daily basis. On your physical exam, you confirm a urethral prolapse. What is the best treatment option?

a. Penile amputation and prescrotal urethrostomy with concurrent castration
b. Penile amputation and perineal urethrostomy with concurrent castration
c. Apply sugar to the urethral prolapse, start patient on prednisone, and castrate in one week
d. Urethropexy with concurrent castration

A

d. Urethral prolapse is a relatively rare condition in dogs; however, it is most often seen in young male English bulldogs. Although the etiology is unknown, it is suspected that sexual excitement, masturbation, genitourinary infection, and calculi may play a role. Additionally, a genetic component is likely present, since English bulldogs are over-represented; therefore, concurrent castration is always recommended and thought to decrease the incidence of recurrence.

There are several treatment options for urethral prolapse, including placing a purse string suture, amputation of the prolapsed region, and urethropexy. The urethropexy technique (Kirsch JAAHA 2002) is (although not proven) thought to be associated with the least likelihood of recurrence. Although an amputation of the penis can be performed, this would be excessive for a urethral prolapse and therefore is not recommended due to to urethrostomy-associated complications. Attempting medical management is likely to result in recurrence, so it is generally not recommended.

108
Q

A 5-year old female Golden Retriever presents for lethargy, dark runny stool, and dehydration. Bloodwork from yesterday showed markedly elevated plasma endogenous ACTH levels, K+ = 6.2 (3.9-5.1 mEq/L), Na+ = 135 (142-152 mEq/L), BUN = 62 mg/dl (8-28 mg/dl). Long-term maintenance therapy for this patient should most likely include:

a. Supplementation with physiologic doses of prednisone
b. Mineralocorticoid, glucocorticoid, and NaCl supplementation
c. IV fluids and daily doses of IV dexamethasone sodium phosphate
d. Mineralocorticoid supplementation alone

A

b.The correct answer is mineralocorticoid, glucocorticoid, and NaCl supplementation. This dog has hypoadrenocorticism which is usually a deficiency in both glucocorticoids and mineralocorticoids. Elevated endogenous plasma ACTH concentrations with hypoadrenocorticism means the disease is due to primary adrenocortical insufficiency and failure to produce both types of corticosteroids (as opposed to secondary adrenocortical insufficiency, when the pituitary does not produce sufficient ACTH). Maintenance treatment of this disease includes supplementation with mineralocorticoids (e.g. Fludrocortisone acetate), glucocorticoids (e.g. Prednisone), and sodium chloride in the diet. IV fluids and dexamethasone sodium phosphate is used in an acute crisis.

109
Q

A 6-month old male Newfoundland presents for a physical exam. Cardiac auscultation reveals a 3/6 left systolic murmur at the heart base. Further diagnostics reveal the diagnosis of mild to moderate subaortic stenosis. What should you tell the owner?

a. The prognosis is poor. Most cases of mild to moderate aortic stenosis do not live past 4 years of age despite all therapy.
b. The prognosis is fair. Sudden death may occur at any time, and the dog has an increased risk for infective endocarditis.
d. The prognosis is relatively good. Balloon dilation of the stenotic region is the treatment of choice and has a high success rate.
e. The prognosis is good if the dog has not developed congestive heart failure yet. Prophylactic treatment with furosemide for congestive heart failure should be prescribed.

A

The correct answer is the prognosis is fair. For dogs with very mild subaortic stenosis, the prognosis is good. Dogs with moderate or severe disease, however have a much more guarded prognosis. Sudden death may occur at any time, and the dog has an increased risk for infective endocarditis. Ventricular arrhythmias can occur and may be the factor leading to sudden death in these patients. Animals with aortic stenosis are at increased risk for infective endocarditis and should be placed on prophylactic antibiotics whenever they undergo a surgical procedure, dental procedure, get an open wound, or experience anything that may potentially lead to bacteremia. Balloon dilation does not improve the prognosis for dogs with subaortic stenosis, though does improve the prognosis for dogs with pulmonic stenosis. Starting furosemide before the onset of congestive heart failure is not advised.

110
Q

A 9 year old male Labrador Retriever presents for acute onset of circling to the right, head tilt to the right, nystagmus to the left, and vomiting. The rest of the physical exam is unremarkable. CBC, chemistry panel, urinalysis, and otic exam were unremarkable. After 14 days of supportive care, the dog has completely recovered on its own. What is the most likely disease this dog had?

Canine idiopathic vestibular disease (Old dog vestibular disease)
Bacterial meningitis
Otitis interna
Inflamed polyp in the middle ear

A

The correct answer is canine idiopathic vestibular disease (Old dog vestibular disease). This disease is usually diagnosed after all other vestibular causes have been ruled out. Otitis interna would be ruled out with an otic exam, CBC, and lack of pain and fever. Bacterial meningitis could be ruled out by a CBC and lack of pain and fever. Polyps in the ear rarely occur in dogs; most aural polyps occur in cats. In addition, an otic exam would rule this out anyway.

111
Q

Pheochromocytomas are not very common in dogs; however, their presence can result in hypertension, tachyarrhythmias, seizures, and collapse. What is the substance secreted by this tumor?

 Estrogen
 Calcium
 Catecholamines
 Cortisol
 Aldosterone
A

A pheochromocytoma is a tumor of the adrenal medulla. To review, the adrenal gland is composed of the adrenal cortex (outer layer) which is divided into the zona glomerulosa, zona fasciculata, and zona reticularis.

The zona glomerulosa is the main site of aldosterone production. The zona fasciculata produces glucocorticoids (mainly cortisol). The zona reticularis is the site of androgen production.

The adrenal medulla is the inner part of the adrenal gland and consists of chromaffin cells. These cells are responsible for making catecholamines (epinephrine and norepinephrine).

112
Q

How is pulmonary thromboembolism definitively diagnosed in a live patient?

Cytology
Blood gas analysis
Contrast radiographs
Plain radiographs

A

The correct answer is contrast radiographs. Angiography is the gold standard for diagnosing PTE. If positive, there are sudden interruptions in blood and contrast flow. Plain radiographs are often normal or have mild changes. Cytology samples would mainly show blood and would not be considered diagnostic or definitive. Blood gas changes typical for PTE are hypoxemia and hypocapnia, but these are not specific for PTE. CT angiography is also a reasonable test to confirm PTE.

113
Q

There are several procedures that are typically performed while correcting medial patellar luxation in dogs. Which one of the following is not performed while repairing a medially luxating patella?

Medial release of the soft tissues
Medial transposition of the tibial tuberosity
Block recession of the trochlear groove
Lateral imbrication of the retinaculum

A

The correct answer is medial transposition of the tibial tuberosity. If you have a medially luxating patella, you need to transpose the tibial tuberosity laterally in order to line up the patellar tendon with the rest of the stifle in an effort to reduce the likelihood of patellar luxation. The two most important procedures that reduce the incidence of recurrence are lateral transposition of the tibial tuberosity and modifying the trochlear groove of the femur.

114
Q

Which of the following is not used as a medical management option in congestive heart failure in a dog?

Enalapril
Lidocaine
Furosemide
Sodium restricted diet

A

The correct answer is lidocaine. Lidocaine is used for treating ventricular arrhythmias. Sodium restricted diets, ACE inhibitors such as enalapril, and diuretics such as furosemide decrease blood volume and preload. ACE inhibitors also reduce afterload by preventing production of angiotensin-II, a potent vasoconstrictor. Pimobendan is also beneficial in the management of congenstive heart failure in the dog due to its positive inotropic effects.

115
Q

A dark red vaginal discharge in a bitch 3 weeks after whelping indicates what process?

 Retained placentas
 Normal response
 Pyometra
 Subinvolution of placental sites 
 Metritis
A

The correct answer is normal response. Dark red vaginal discharge occurs normally for up to 6 weeks after whelping. Subinvolution can cause fresh bleeding for 12-15 weeks post-partum. Metritis is usually accompanied by systemic signs and may have abnormal vaginal discharge but not usually hemorrhage. Retained placentas typically do not occur in bitches. Pyometras occur during the luteal phase when progesterone levels are high, up to 9 weeks after estrus, and would not occur post-whelping.

116
Q

Dolly, a 3-year old female spayed Siamese mix, presents with a history of weight loss over the last month. She is now vomiting occasionally and has a decreased appetite over the last 2 weeks. She has not eaten in the last 2 days. She is current on vaccines. Temperature is 103.7 F (39.8 C). You can palpate an abdominal mass effect in the mid to caudal abdomen. You believe the cat has a foreign body and are concerned about a possible intestinal perforation. Abdominal tap is negative. Pre-anesthetic bloodwork shows neutrophils 25,000 /ul (2,500-12,500/ul), bands 3,000 /ul (0-300/ul), globulins 6.9 g/dL (2.6-5.1 g/dl). Your x-ray machine is not working today and you recommend an abdominal exploratory. Upon exploratory, the intestines are severely hyperemic and the mesenteric lymph nodes are greatly enlarged. You cannot find a foreign body and no perforations are seen. There is a small amount of yellow tinged sticky ascites. What do you do?

a. Biopsy the lymph node and intestine and discuss a poor prognosis with the owner
b. Euthanize the cat on the table since you are unable to reach the owner over the phone about the poor prognosis
c. Start the cat on prednisolone and hypoallergenic diet for severe inflammatory bowel disease
d. Perform a fecal flotation and start sulfadimethoxine for a severe coccidial infection
e. Refer to an oncologist for work up of gastrointestinal lymphoma

A

This cat most likely has Feline Infectious Peritonitis, or FIP, which is caused by a mutation of a feline corona virus. Fever, weight loss, and gastrointestinal symptoms are the most frequent presentation. However, this virus may attack multiple organs and can be difficult to diagnose; the only definitive way to diagnose this disease is via histopathology. Clinical symptoms, blood results, and corona virus titers can all be used in combination to help aid in the suspected diagnosis of FIP. Unfortunately, there is no cure for this disease and it is currently considered a fatal disease.

Due to this cat’s declining and critical health and strong evidence to support the diagnosis of FIP, euthanasia may be the most humane option. If the owner cannot be reached, however, it is the best option to go ahead and take biopsies while you are in surgery and then discuss the prognosis and differentials in detail when you can reach the owner. An animal should not be euthanized without owner consent.

Inflammatory bowel disease should not cause ascites and fever.
Parasites can lead to ascites, but typically would not cause the elevations of white blood cells and globulins with fever.
While lymphoma may be a possibility, it is less likely in this young cat. The biopsies would help to differentiate.

117
Q

You confirmed chylous pleural effusion in your feline patient and referred to a specialist. The owner wants to discuss treatment options with you. The cat has had an extensive workup and no underlying cause has been found. Medical management has been tried for the last 3 months to no avail. The cat has been on a low fat diet and Rutin with intermittent thoracocentesis when needed. Which of the following treatments would be the best option and be most likely to resolve the effusion?

Ligation of thoracic duct and pericardectomy
Place a chest tube to keep drained consistently for 1 week and this should resolve the fluid
Vitamin E and milk thistle supplementation
Chemotherapy
Somatostatin

A

Surgery is the treatment of choice if medical therapy is failing. The best chance for resolution of a chylous effusion is ligating the thoracic duct and pericardectomy. Even with surgery, the effusion can still continue but this is the best chance for a cure.

Somatostatin is a naturally occurring substance in gastric, pancreatic, and biliary secretions. In recent years, analogues of somatostatin have been used to successfully treat chylothorax in humans. The mechanism by which non-traumatic chylothorax may benefit from this treatment is unclear; however, resolution of pleural fluid (chyle and postoperative serosanguineous effusion) in both dogs and cats has occurred after administration of octreotide (somatostatin). It is extremely expensive and not as likely to cure the effusion as surgery would.

Vitamin E and milk thistle supplements are sometimes used in cases of liver disease but not indicated for chylous effusion.

Despite chest tube placement to keep the fluid drained, the effusion will continue to occur because the drain does not stop the fluid from being produced.

Chemotherapy would not be indicated in a case of idiopathic chylous effusion.

118
Q

A 5-year old male castrated English Springer Spaniel presents for a dental exam. The dog has a history of aggressive chewing on tennis balls and frisbees. The oral exam reveals brown, worn-down incisors, canines, and premolars. The dog is otherwise normal and healthy. What is the cause for the teeth turning brown?

a. Excessive chewing has predisposed the teeth to infection. The brown coloration is a sign of bacterial infection.
b. The dog was probably given tetracycline antibiotics as a puppy, which causes a permanent brown discoloration of teeth.
c. The excessive chewing has worn away the enamel of the teeth. The underlying dentin is naturally brown.
d. The teeth are brown due to the formation of tertiary dentin, which stains easily.

A

The correct answer is the teeth are brown due to the formation of tertiary dentin, which stains easily. Aggressive chewing of things like rocks, tennis balls, cage bars, etc. causes abrasion of dentin. Tertiary dentin, which is produced as a response to the wearing of the teeth, stains easily. Tertiary dentin will usually prevent the exposure of the pulp cavity unless its production cannot keep up with its rapid wear.

119
Q

It is a busy day in the clinic and you are down to your last appointment. You are excited to leave for the day and can’t wait to finish this general examination of a 1-year old female cat that was just adopted from the local rescue. The owners would like a general health screen along with information on spaying the cat. On physical exam, you detect a continuous machinery murmur, which is audible on both sides of the chest. Otherwise, the examination was unremarkable, pulses were strong and symmetrical, normal temperature, heart rate, and respiratory rate. The cat was purring making it difficult to hear the murmur but you are certain it is present. You discuss with the owners the likelihood of a patent ductus arteriosus and recommend an echocardiogram to confirm the diagnosis. You also explain to the owners that this condition, although seen in cats, is much more commonly seen in dogs. Assuming the owners will go on to have surgery performed, what muscular landmark allows the surgeon to determine he/she is at the 5th rib space?

 Superficial pectoral muscle
 Cutaneous trunci muscle
 Scalenus muscle
 Latissimus dorsi muscle
 Serratus ventralis muscle
A

The correct answer is scalenus m. This applies to both dogs and cats. The 5th rib marks the end of the muscular portion of the scalenus and the beginning of the external abdominal oblique. It is almost impossible to remember all the origins and insertions of muscles; a good tip is to try and at least remember those that provide important surgical landmarks.

The scalenus lies ventral to the origin of the cervical and thoracic parts of the serratus ventralis. It attaches to the first few ribs and the transverse processes of the cervical vertebrae and inserts on the 5th rib.

The serratus ventralis is fan-shaped and originates on the transverse processes of the last five cervical vertebrae and the first seven or eight ribs. It inserts on the scapula.

The superficial pectoral muscle originates on first two sternebrae and usually part of the third. It goes on to insert on the whole crest of the greater tubercle of the humerus.

The latissimus dorsi covers most of the dorsal and some of the lateral thoracic wall. Its origin is at the spinous processes of the lumbar and last 7 or 8 thoracic vertebrae. It inserts on the teres major tuberosity of the humerus and teres major tendon.

The cutaneous trunci muscle is a thin sheet of muscle that covers most of the dorsal, lateral, and ventral walls of the thorax and abdomen. This muscle is responsible for twitching the skin and is innervated by the lateral thoracic nerve.

120
Q

A 10 year old female spayed German shepherd dog presents for collapse and difficulty breathing. Your physical exam shows pale mucous membranes, a fluid wave in the abdomen and a splenic mass. Aspiration of the abdominal fluid shows hemorrhagic effusion that does not clot. Which of the following is commonly found with your presumed diagnosis?

 Factor VIII deficiency
 Tumor lysis syndrome
 Hypercalcemia
 Disseminated intravascular coagulation 
 Von Willebrand's disease
A

The presumed diagnosis based on the given signalment, physical findings, and diagnostics is splenic hemangiosarcoma, which is often associated with DIC.

121
Q

Which is a reasonable plan for the treatment of bacterial pneumonia in a dog?

a. Systemic antibiotics, cough suppressants, coupage, oxygen therapy
b. Oxygen therapy, diuretics, bronchodilators, and systemic antibiotics
c. Systemic antibiotics, bronchodilators, corticosteroids, oxygen therapy
d. Coupage, systemic antibiotics, oxygen therapy, and nebulization

A

The correct answer is coupage, systemic antibiotics, oxygen therapy, and nebulization. All of the choices include systemic antibiotics and oxygen therapy, which are absolutely indicated. Coupage and nebulization are also helpful adjuncts. Coupage may mechanically jar secretions and stimulate cough and can be performed for 5-10 minutes several times daily. Nebulization is to maintain airway hydration. It is not always necessary but is appropriate. Cough suppressants are contraindicated as are corticosteroids since they both interfere with normal defenses. Diuretics are contraindicated as well as they decrease airway hydration. Bronchodilators are controversial but are not a mainstay of therapy for pneumonia unless bronchoconstriction is present.

122
Q

Which are the most common serovars now thought to play a role in canine leptospirosis?

a. Grippotyphosa, pomona, bratislava
b. Bratislava, canicola, icterohemorrhagiae
c. Pomona, bratislava, icterohemorrhagiae
d. icterohemorrhagiae, canicola, grippotyphosa

A

a. The correct answer is grippotyphosa, pomona, and bratislava. Icterhemorrhagiae and canicola were the most common serovars isolated in the past. The leptospires penetrate mucous membranes or abraded skin and multiply in the blood stream and spread to organs. The antibody response usually limits the response to the renal tubular epithelial cells. Clinical signs include anorexia, pyrexia, vomiting, dehydration, PU/PD, anuria or oliguria. The standard diagnostic test is the microscopic agglutination test (MAT); be careful when interpreting titers. Remember, leptospirosis is zoonotic.

123
Q

7yo M Shar-Pei. lethargic, PU/PD, anorectic.
dec. WBC, HCT 29%, Azotemia, urinalysis 1.010, 4+ protein.

The dog has only a minimal response to supportive care with IV fluids, gastric protectants, and pain medications. He is euthanized after 2 days of hospitalization. On postmortem exam, you find that his kidneys are grayish in color and enlarged. The cut surface is firm and waxy. What it your presumptive diagnosis?

 Amyloidosis 
 Lyme disease
 Hydronephrosis
 Glomerulonephritis
 Renal infarct
A

a. The marked proteinuria suggests glomerular disease. The necropsy finding of a waxy kidney is highly suggestive of amyloidosis. Shar-Pei dogs are predisposed to this condition.

In hydronephrosis, you would see extensive dilation of the renal pelvis and calyces at postmortem. With renal infarcts, you see triangular or pyramidal shaped lesions. Lyme disease can lead to glomerulonephritis where the surface of the kidney is covered by hemorrhagic spots.

124
Q

A 2-year old Golden Retriever presents to you after jumping from a height of 15 feet and sustaining a severe carpal hyperextension injury in his right carpus. The treatment of choice for this dog is _________.

a. Non-steroidal anti-inflammatory drugs and cage rest for 3 weeks
b. Carpal arthrodesis
c. Splint and cage rest for 3 weeks
d. Cast for 6-8 weeks

A

b. The correct answer is carpal arthrodesis. Arthrodesis is the treatment of choice for severe hyperextension injuries to the carpus. This procedure is accomplished by debridement of the articular cartilage of the joints, implantation of a cancellous autograft into the debrided joint spaces, and fixing a bone plate across the injured joints

125
Q

A male neutered domestic short hair cat presents to you with urinary obstruction. Your treatment room diagnostics show a BUN > 140 mg/dL (19-34 mg/dl) and a K+ of 9.0 mEq/L (3.7-6.1 mEq/L). You immediately give the cat dextrose and fluids and successfully unblock your patient. An hour later, you monitor your patient and find him trembling uncontrollably. What should you check first?

A blood pressure
An ionized calcium level
A spot ECG
A K+ level

A

In some cases, during the immediate post-obstructive period, cats will develop hypocalcemic tetany. The mechanism is considered to be secondary to laws of mass action that drive down calcium as a result of existing hyperphosphatemia. Judicious use of calcium gluconate can quickly restore their electrolyte balance and prevent further complications such as hypocalcemic seizures.

126
Q

A 6-month old male German Shepherd Dog presents for castration. A 4/6 left systolic murmur is heard best over the apex of the heart. Thoracic radiographs reveal a moderately enlarged left atrium. What is the most likely diagnosis?

Mitral valve dysplasia
Aortic stenosis
Myxomatous degeneration of the mitral valve
Tricuspid valve dysplasia

A

a. The correct answer is mitral valve dysplasia. This abnormality is a congenital anomaly of the mitral valve. The valve leaflets may be thickened, fused, fibrosed, etc. The chordae tendinae or the papillary muscles attaching to the mitral valve may be irregular as well. German Shepherd Dogs, Great Danes, and other large breed dogs are predisposed to this disease. Myxomatous mitral valve degeneration occurs in older dogs, but would otherwise result in similar physical exam and radiographic findings as those described. Tricuspid dysplasia results in an enlarged right heart, and the murmur would be heard best on the right side of the dog. Aortic stenosis results in concentric hypertrophy of the left heart, which may or may not be detectable on radiographs. Also, the murmur is heard best at the left heart base with aortic stenosis.

127
Q

Which of the following statements is true about degenerative disc disease in dogs?

a. Hansen’s Type 1 disc disease usually involves chondrodystrophic dogs. It is usually chronic, non-painful, and is not considered an emergency.
b. Hansen’s Type 2 disc disease usually involves non-chondrodystrophic dogs. It is usually chronic, painful, and should be considered an emergency.
c. Hansen’s Type 2 disc disease usually involves non-chondrodystrophic dogs. It is usually acute, painful, and should be considered an emergency.
d. Hansen’s Type 1 disc disease usually involves chondrodystrophic dogs. It is usually acute, painful, and should be considered an emergency.

A

d. The correct answer is Hansen’s Type 1 disc disease usually involves chondrodystrophic dogs (Dachshunds, Corgis, Shih-Tzus, etc.). It is usually acute, painful, and should be considered an emergency.

128
Q

How much protein should you feed a dog with hepatic insufficiency or hepatic encephalopathy?

a. The total quantity of protein is not important as long as they eat predominantly meat proteins rather than dairy proteins
b. The maximum amount of protein they will tolerate without causing signs of encephalopathy
c. The minimum amount of protein they will tolerate without developing hypoproteinemia
d. 1.0 gram protein/kg/day

A

b. Dogs with hepatic insufficiency or hepatic encephalopathy need to have their protein levels restricted to reduce clinical signs associated with liver dysfunction; however, restricting protein intake to the point where hypoproteinemia develops is excessive. If too little protein is fed to these patients, muscle catabolism may occur. Proteins derived from meat sources are more prone to causing encephalopathy than proteins from dairy sources due to the high levels of nucleic acids and other nitrogenous compounds found in meat that are converted to ammonia.

129
Q

A 6 year old female spayed Labrador Retriever presents for a progressive lameness of the left thoracic limb over the past 3 months. Physical exam findings include no conscious proprioception, no withdrawal reflex, and atrophy of the muscles of the affected limb. An ultrasound of the left axilla shows a soft tissue mass lesion extending up from the axilla to the vertebral canal. What is the most likely diagnosis?

Osteosarcoma
Hemangiosarcoma
Peripheral nerve sheath tumor
Fibrosarcoma

A

c. The correct answer is peripheral nerve sheath tumor. The presentation of the dog described is classic for a peripheral nerve sheath tumor. They are slow growing tumors arising from the peripheral nerve sheath cells. They are most commonly found in the brachial plexus. Limb amputation and tumor excision may be curative if the spinal canal has not been invaded.

130
Q

What is the typical ultrasonographic appearance of the abdomen in a dog with pancreatitis?

a. There is a hyperechoic pancreas with hypoechoic surrounding mesentery
b. There is a hypoechoic pancreas with hypoechoic surrounding mesentery
c. There is a hypoechoic pancreas with hyperechoic surrounding mesentery
d. There is a hyperechoic pancreas with hyperechoic surrounding mesentery

A

The correct answer is there is a hypoechoic pancreas with hyperechoic surrounding mesentery. Sometimes, the pancreas will also be enlarged and may appear mottled. The pancreas appears hypoechoic due to edema, and the mesentery appears hyperechoic due to focal peritonitis.

131
Q

You examine a Basset Hound with primary glaucoma in one eye. What would you tell the owner about his prognosis for the other eye?

a. It will probably also develop glaucoma in 6-12 months
b. It has no increased chance of developing glaucoma
c. It will probably develop glaucoma within the next 1-3 months
d. This is very strange because glaucoma almost always occurs bilaterally

A

The correct answer is that it will probably also develop glaucoma in 6-12 months. The usual course for primary glaucoma is development in one eye with the contralateral eye following in 6-12 months. These animals have an iridocorneal angle that becomes increasingly compromised during the first few years of life and eventually causes an acute pressure spike in the eye.

132
Q

Currently, there is debate regarding meniscal releases in canine patients with cranial cruciate ligament disease. Which of the following is most accurate?

a. The medial meniscus is a highly vascular structure and receives only about 5% of its nutrition from the surrounding synovial fluid
b. There is a less than 7% incidence of post-operative medial meniscal injury rate after a tibial plateau leveling osteotomy
c. The bucket handle tear of the medial meniscus is the rarest form of meniscal tears
d. Medial meniscus injury has been reported to occur in less than 3% of dogs with cranial cruciate ligament disease

A

b. Studies show a post-operative meniscal injury rate ranging between 2-7% when performing a tibial plateau leveling osteotomy. This is in contrast to a study on the tibial tuberosity advancement procedure which had a much higher incidence of post-operative medial meniscal tears. The two most common meniscal injuries observed are the bucket handle tear (caudal longitudinal tear) and a “crush” of the caudal horn of the meniscus (this is essentially an incomplete bucket handle tear). The outer 25-30% of the meniscus is avascular which helps explain why meniscal injuries don’t heal well. Additionally, the inner 2/3 of the meniscus is also avascular.

This level of detail is probably greater than what you will need to know for a board examination but it is important to understand that performing a meniscal release at the time of cruciate ligament surgery may not be indicated based on the low probability of post-operative injury to this structure.

133
Q

In dogs, large doses of epinephrine cause which of the following?

a. Increase in arterial blood pressure, increase in cardiac output, bronchoconstriction
b. Decrease in total peripheral resistance, increase in cardiac contractility, bronchodilation
c. Increase in arterial blood pressure, decrease in cardiac contractility, bronchodilation
d. Increase in total peripheral resistance, increase in cardiac contractility, bronchodilation

A

d. The correct answer is increase in total peripheral resistance, increase in cardiac contractility, bronchodilation. Epinephrine is an alpha- and beta-adrenergic stimulant which causes peripheral vasoconstriction, increase in total peripheral resistance, increase in cardiac contractility and cardiac output, and bronchodilation. It is often used in cardiac arrests and anaphylactic shock. It is also found in some local anesthetic agents because of its vasoconstrictive effects, which prolongs effect of the anesthetic.

134
Q

Which of the following problems is seen more frequently in male dogs than females?

 Adenocarcinoma of the anal sac
 Urinary tract infections
 Mammary cancer
 Aggression
 Addison's disease (hypoadrenocorticism)
A

More males than females present to veterinary behaviorists for aggression. The other diseases listed have been reported or suggested to occur more frequently in females in some reports.

135
Q

Which of the following parasites that can be found on dogs is not contagious?

 Trichodectes
 Cheyletiella
 Otodectes
 Demodex
 Sarcoptes
A

The correct answer is Demodex. Demodex is an normal inhabitant of the skin and causes disease when there is either a genetic predisposition or systemic disease that allows Demodex to proliferate, resulting in skin pathology. Therefore it is not contagious in dogs. Demodex gatoi in cats is thought to be contagious amongst cats.

136
Q

A 9-year old male, neutered German Shepherd dog is presented to your clinic for evaluation of tenesmus, tail chasing, and hematochezia. Malodorous mucopurulent discharge is noted near his anus. On examination, you find multiple ulcerations around his perineum. Rectal examination reveals reduced anal tone and mucosal thickening. What are your treatment recommendations for this dog?

a. Initiate broad-spectrum antibiotics to treat for an infected anal gland abscess
b. Initiate chemotherapy with the alkylating agent, melphalan, as this is the treatment of choice for anal sac adenocarcinoma
c. Initiate therapy with cyclosporine
d. Surgically correct with a 360-degree anoplasty
e. Initiate therapy with metronidazole

A

The signalment, history and clinical signs are most consistent with a perianal fistula. Although the pathogenesis is not fully understood, this disease is thought to be primarily immune-mediated. The treatment of choice for perianal fistula is medical management with cyclosporine, as resolution is achieved in greater than 65% of dogs. Surgical correction may lead to fecal incontinence or stricture formation, however surgery for removal of the anal sacs in combination with cyclosporine has recently shown promise.

If this was an anal gland abscess, lancing the abscess and drain placement with systemic antimicrobial therapy would be indicated. If you suspected this to be a cancerous process, a minimum database including CBC, chemistry, thoracic radiographs and abdominal ultrasound are recommended for disease staging. The treatment of choice for anal sac adenocarcinoma is surgical resection, followed by adjunct therapy.

137
Q

A 5 year old male castrated Welsh Corgi presents for acute paralysis of its pelvic limbs. The owners note that the dog was fine in the evening, but became paralyzed in the pelvic limbs in the morning after it jumped off of the owner’s bed. The dog has hyper-reflexive spinal reflexes and no motor function or deep pain in the pelvic limbs. CBC, chemistry panel, and urinalysis are unremarkable. Survey spinal radiographs show a narrowed disc space between T11-T12. What is the next appropriate step for this dog?

Cage rest and NSAIDs
Ventral slot surgery
Dorsal hemilaminectomy
Contrast myelogram and decompression surgery

A

The correct answer is contrast myelogram and decompression surgery. The dog has Hansen’s Type 1 degenerative disc disease. The survey radiographs are enough to diagnose the disc rupture, but myelography should be performed to better assess the areas of the spinal cord affected and to definitively identify which side should be approached for decompression surgery. The lack of deep pain is a poor prognostic indicator, so cage rest and NSAIDs are not adequate in this patient.

138
Q

Pookie, a 2 year-old female spayed poodle mix, was presented to your hospital for acute severe vomiting and bloody diarrhea. The owner notes that Pookie recently got into the trash. On physical exam, you find Pookie is dehydrated, uncomfortable on abdominal palpation, and has hematochezia on rectal exam. You run a complete blood count and chemistry panel and find a PCV of 60% (36-50%) and total protein of 6.0 (5.5-7.3 g/dL). A urinalysis is unremarkable. Abdominal radiographs show no evidence of obstruction. A fecal float is negative. What is the most important treatment for the suspected diagnosis for this patient?

 Fenbendazole
 Enrofloxacin 
 Maropitant
 Intravenous fluids
 Famotidine
A

Pookie’s history, physical exam, and lab work are consistent with hemorrhagic gastroenteritis (HGE), a severe form of gastrointestinal disease that is very common among small dogs. Aggressive fluid therapy is extremely important for these patients to prevent shock, disseminated intravascular coagulation, and renal failure. An abnormally high PCV can be a good indicator of this disease if there is a history of blood diarrhea. Anti-nausea and gastroprotective medications may help the patient to feel better, but aggressive fluid therapy is the most important treatment for these patients. Maropitant is an anti-emetic. Famotidine is a gastroprotectant used to raise the pH in the stomach. Fenbendazole is a dewormer medication. Enrofloxacin is a fluoroquinolone antibiotic.

139
Q

You are examining a 3-year old female Miniature Poodle whose owner complains that she has been bumping into things more and more recently, especially at night. On fundic examination, you note tapetal hyperreflectivity, gray, vermiform lines on the fundus, retinal vascular attenuation, and a pale optic disc. Which of these choices is the most likely cause of her problem?

Sudden Acquired Retinal Degeneration Syndrome (SARDS)
Glaucoma
Taurine Deficiency
Progressive Retinal Atrophy (PRA)

A

The correct answer is PRA. Progressive retinal atrophy, sometimes called progressive retinal degeneration is an inherited retinal disease seen in several breeds but most notably Toy and Miniature Poodles. Age of onset is variable, but signs are usually night blindness progressing to complete blindness due to loss of rods prior to cones. Retinal lesions are classically the ones described in this case. It is differentiated from SARDS by the clinical course and the fact that there are no retinal lesions in acute SARDS. You could see similar retinal lesions from glaucoma but would expect more clinical signs and you wouldn’t have night blindness progressing. Taurine deficiency causes retinal lesions in cats not dogs, and they have a different appearance.

140
Q

For what type of surgery would nitrous oxide be contraindicated for use as part of an anesthetic protocol in a dog?

 Splenic surgery
 Hepatic surgery
 Gastrointestinal surgery 
 Open chest surgery
 Renal surgery
A

GI
Nitrous oxide moves into closed gas spaces such as the intestines. As such, its use is contraindicated in bowel surgeries.

You should also be aware that nitrous oxide decreases fractional inspired oxygen levels although this can be managed and monitored in most instances. Nitrous oxide is contraindicated when pathology such as pulmonary bullae are present.

141
Q

Which of these is a correct description of how lactulose works in the treatment of hepatic encephalopathy?

It slows down the passage of food, leading to more gradual uptake of ammonia.
It acts centrally to decrease the neuromodulatory effects of ammonia.
It causes decreased colonic pH, preventing ammonia absorption. Correct Answer
It kills ammonia producing bacteria in the colon.

A

The correct answer is that it decreases colonic pH preventing absorption of ammonia. Lactulose is an easily fermented carbohydrate that is metabolized to an acid in the gut. This lowers colonic pH, which keeps ammonia in its ionized form, NH4+, rather than as NH3. The ionized form is not absorbed and is excreted. Lactulose also acts through a number of other mechanisms. It is a cathartic and causes decreased intestinal transit time, leading to decreased absorption of ammonia. It also prevents ammonia formation through a process known as catabolite repression. Finally, it is a carbohydrate source which can be used by colonic flora as an alternative to protein, preventing ammonia formation.

142
Q

Which of the following is most likely to infect and cause dermatitis in a child walking barefoot through a field?

 Toxocara spp.
 Ancylostoma spp.
 Trichuris vulpis
 Dipylidium caninum
 Dirofilaria immitis
A

The correct answer is Ancylostoma spp. Ancylostoma caninum, the dog hookworm, can infect a human by penetrating skin. It can cause dermatitis via cutaneous migration of the parasite. Toxocara is a roundworm that causes visceral larval migrans.

143
Q

Which of the following is the most likely cause of lymphoplasmacytic rhinitis as seen in this patient?

a. The specific etiology is still unknown
b. Bartonella
c. Parainfluenza virus-3
d. Canine adenovirus-2

A

As the name implies, these patients have a mixed population of lymphocytes and plasma cells when biopsied. Many consider the condition to be an idiopathic immune-mediated disorder. PCR studies have been performed in affected patients in search of parainfluenza virus-3, canine adenovirus-2, Bartonella, and Chlamydophila. None of these agents were found to be a potential cause. Typical clinical signs include a history of unilateral or bilateral nasal discharge of several months duration. Large-breed dogs are more often affected and the age may vary.

144
Q

Treatment of choice for proptosis?

A

The answer is tarsorrhaphy. Tarsorrhaphy is the treatment of choice for proptosis. In this procedure, the eyelids are pulled in front of the proptosed globe rather than pushing the eye in and then temporarily suturing the lids shut. Exenteration is a procedure used to remove eyes with neoplasia that is infiltrating into the surrounding structures. A conjunctival flap is a procedure usually used to help healing of complicated corneal ulcers. Phacoemulsification is the procedure used to remove cataracts.

145
Q

A 3-year old Golden Retriever presents for acute onset of a non-painful myelopathy localized to the spinal segments from T3-L3. The dog has superficial and deep pain perception and some motor function present in its pelvic limbs. It also has hyperreflexive patellar and gastrocnemius reflexes. The dog is diagnosed with a fibrocartilagenous embolism (FCE). What do you tell the owners?

a. Most dogs with FCE usually get better on their own with nursing care
b. The dog should be treated with heparin to prevent further emboli
c. The dog will probably continue to get worse and has a poor prognosis
d. The dog should have decompressive surgery

A

The correct answer is most dogs with FCE usually get better on their own with nursing care. FCE is caused by an embolus of disc material in the spinal vasculature. It is unknown how the disc material gets into the vasculature. The prognosis of dogs with FCE varies. Most dogs get better with supportive and nursing care. The loss of pain perception and lower motor neuron signs are poor prognostic indicators.

146
Q

A 4-year old female spayed Australian Shepherd presents for recurrence of left thoracic limb lameness. The dog has a history of a left radial fracture 12 weeks ago, which was repaired with a bone plate and has since fully healed. Over the past three days, the dog started limping on the left thoracic limb and has developed a draining tract over the site of the previous fracture repair. Radiographs of the limb show soft tissue swelling and lucencies around the bone plate and several of the screws. What should be done next?

a. Remove the screws with surrounding lucencies, replace them with new screws, and start antibiotics
b. Remove the bone plate and start antibiotics Correct Answer
c. Remove one of the screws with lucent surroundings for culture and sensitivity, and start antibiotics after sensitivity results come back
d. Start antibiotics

A

The correct answer is remove the bone plate and start antibiotics. Implants such as bone plates and screws can often be a nidus for infection. The lucencies around the bone plate and screws suggest instability of the implants, and the draining tract is suggestive of infection. Since the bone is already fully healed, the best treatment for this dog would be to remove the bone plate and screws and start antibiotics.

147
Q

A 7-year old female spayed Golden Retriever that you suspect has Addison’s disease has bloody diarrhea, inappetance, and is dehydrated. What should be your next step?

a. Start aggressive IV fluid therapy and run a chemistry panel
b. Run a chemistry panel and perform an abdominal ultrasound.
c. Give Lysodren
d. Give IV dexamethasone and subcutaneous fluids

A

The correct answer is start aggressive IV fluid therapy and run a chemistry panel.

Addisonian patients often present in hypovolemic shock, so the first step to treating this dog is to restore vascular volume with IV fluids and to run a chemistry panel to check the extent of electrolyte abnormalities (hyponatremia, hyperkalemia, and elevated BUN). IV dexamethasone would not take precedence over restoring vascular volume, and subcutaneous fluid treatment is not aggressive enough for treatment of hypovolemia. Abdominal ultrasound would not be a priority in this case but could eventually be helpful in ruling in/out other differentials.

148
Q

Which of these is most likely to provide a definitive diagnosis that a dog with chronic nasal discharge has nasal aspergillosis?

 Serology
 Fungal culture
 Cytology
 Histopathology 
 CT scan
A

The correct answer is histopathology. Diagnosis of nasal aspergillosis is based on demonstrating the organism in diseased tissue. A CT is great for looking at turbinate destruction but can look very similar if there is a nasal tumor. Cytology usually does not reveal fungal hyphae. Fungal culture is very unreliable; about one third of dogs with nasal tumors may have growth on fungal culture. Serology is also riddled with false negatives and positives. Histopathology usually reveals pyogranulomatous inflammation and necrosis with numerous fungal hyphae. The yield of biopsies can be greatly improved with assistance from rhinoscopy where white fungal plaques are often visualized. Diagnosis is sometimes made from rhinoscopy alone although this is less definitive than histopathology.

149
Q

Which of the following is a common finding on a complete blood count in a dog with a ruptured splenic hemangiosarcoma?

 Thrombocytosis
 Neutropenia
 Eosinophilia
 Schistocytes 
 Heinz body anemia
A

Schistocytes are fragmented red blood cells that are mechanically broken up due to the irregular vessels and fibrin strands they pass through in hemangiosarcoma patients. Other typical findings on a CBC in a patient with hemangiosarcoma are thrombocytopenia, anemia (although not with Heinz bodies), and leukocytosis.

150
Q

A 4-year old female spayed German Shepherd Dog presents to your clinic for a 3 day onset of progressive, symmetric, ascending paralysis, which started at the pelvic limbs and is now beginning to work its way up the thoracic limbs. You notice the paralysis is flaccid with decreased muscle tone, decreased to absent reflexes, and cranial nerve involvement. Pain perception and mental attitude are normal. What of these choices is the most likely differential diagnosis?

Clostridium botulinum Correct Answer
Tick paralysis Your Answer
Myasthenia gravis
Clostridium tetani

A

The correct answer is C. botulinum. With C. tetani you would see stiffness. Tick bite paralysis can be put lower on the list of likely differentials because there is cranial nerve involvement.

An alternative differential for a presentation such as this one is Coonhound paralysis (idiopathic polyradiculoneuritis) may be seen after a raccoon bite, systemic illness, or vaccination; the cause is often unknown. Cranial nerve involvement is usually limited to the facial and pharyngeal/laryngeal region. Additionally, diffuse hyperesthesia may be present with Coonhound paralysis.

151
Q

A 12-year old male neutered Terrier mix presents for a geriatric exam. Physical exam reveals mildly prominent mandibular, prescapular, and popliteal lymph nodes, and a prominent spleen.

CBC results: 
hematocrit of 35% (35-57%) 
reticulocytes 45,000/uL (0-100/uL) 
neutrophils 3,000/uL (2,900-12,000/uL) 
lymphocytes 40,000/uL (400-2,900/uL) 
monocytes 900/uL (100-1,400/uL) 
eosinophils 200/uL (0-1,300/uL) 
platelets 120,000/uL (211,000-621,000/uL) 

A bone marrow aspirate shows 40% small lymphocytes. A mandibular lymph node aspirate shows 90% small lymphocytes. What is the cause for these findings?

 High grade lymphoma
 Acute myelogenous leukemia
 Acute lymphoid leukemia
 Chronic myelogenous leukemia
 Chronic lymphocytic leukemia
A

chronic lymphocytic leukemia.

because marrow is affected more than lymph nodes its leukemia over small cell lymphoma (the two are hard to differentiate)

152
Q

A 3-year old German Shepherd presents for multiple subcutaneous nodules on the right front limb that have been present for about one week. The dog is otherwise healthy. The owner states that the dog lives mostly outdoors in a wooded area with ponds in Florida. You are highly suspicious of pythium; what is the best therapeutic option for this disease?

Lufenuron
Antifungal therapy
Ivermectin
Prednisone
Amputation and treatment with doxycycline
Amputation and treatment with an antifungal

A

Pythium is an aquatic organism that inhabits ponds of the Southeast United States. These organisms have different cell walls than regular fungi, which makes them difficult to treat since most antifungal treatments target fungal cell wall synthesis. Prognosis is extremely poor with this disease and amputation (if the lesions are localized to an affected limb) is currently the best option. Amputation should always be followed with multiple long-term antifungal therapies. Owners should be warned that local postoperative recurrence is common. Dogs are most commonly affected, but this disease can also occur in cats. It has also been reported in humans and horses.

Doxycyline is an antibiotic and would not be effective for this disease. Prednisone would be contraindicated for pythium. Lufenuron is a chitin synthesis inhibitor used for flea control and would not be effective. Ivermectin is an anti-parasite drug and would not be effective.

153
Q

A 12-year old female spayed Lhasa Apso presents for her annual physical exam. The owners report that she is doing fine at home. Cardiac auscultation reveals a 2/6 left systolic murmur over the apex of the heart. Thoracic radiographs are within normal limits. Color Doppler echocardiography reveals a small regurgitant jet across a mildly thickened mitral valve. What should be recommended to the owner?

The dog should be placed on diuretic therapy at this time.
The dog should be placed on antibiotics because she is prone to developing bacterial endocarditis with the thickened, irregular valves.
The dog probably has congenital mitral valve dysplasia and has been living with the valve defect her whole life. She does not need medication.
The dog probably has myxomatous mitral valve degeneration and does not need cardiac medications at this time, but she should be rechecked in several months

A

The correct answer is the dog probably has myxomatous mitral valve degeneration and does not need cardiac medications at this time, but she should be rechecked in several months. This disease commonly occurs in older dogs as a result of degeneration and accumulation of acid-staining mucopolysaccharides in the valves (most commonly the mitral valve). A dog with early stage changes does not need to be on cardiac medications. As the disease progresses and the heart chambers enlarge and the animal begins to show signs of heart failure, the dog should be placed on medications such as diuretics, ACE inhibitors, +/- pimobendan. Congenital mitral dysplasia would usually result in more marked cardiac changes (louder murmur, more regurgitation, dilated heart chambers) at a much earlier age. Myxomatous mitral valve degeneration patients do not need to be placed on antibiotics for fear of bacterial endocarditis. Patients with subaortic stenosis are placed on prophylactic antibiotics if put at risk for bacteremia.

154
Q

How would you evaluate whether or not the mandibular branch of cranial nerve V (trigeminal nerve) is functionally intact in a dog?

Look for symmetry and tone of the muscles of mastication
Deviation of the nose to one side
Trigeminofacial reflex
Drooping of the upper lip on one side

A

The correct answer is look for symmetry and tone of the muscles of mastication. The mandibular branch of CN V carries sensory and motor function to the muscles of mastication. If the mandibular branch was not functional, there would initially be decreased tone in the muscles of mastication. After a long period of time, contracture of the muscles would cause an increased tone. The trigeminofacial reflex tests the maxillary branch of CN V (sensory afferent) and CN VII (facial nerve). Deviation of the nose to one side occurs with facial nerve paralysis. The nose deviates away from the lesion when the problem is acute due to lack of muscle tone on that side. The nose deviates toward the side of the lesion when the problem is chronic due to contracture of the muscles on the affected side. Drooping of the upper lip on one side would also be due to a lesion of the facial nerve.

155
Q

A 13-year old spayed female Cocker Spaniel presents to your clinic with the presenting complaint of cough and difficulty breathing. On exam, the dog is subdued but responsive with a temperature of 102.3F (39.1 C), heart rate of 140 beats per minute and respiratory rate of 50 breaths per minute. You also detect a grade IV/VI systolic heart murmur on the left side. You perform chest radiographs which are shown below. (L heart enlargement, some right enlargement) A CBC shows a hematocrit of 44% (35-57%), neutrophil count of 7,115/ul (2,900-12,000/ul), monocyte count of 1,484/ul (100-1,400/ul), and eosinophil count of 430/ul (0-1,300/ul). What is the treatment of choice?

Oxygen, atenolol, and enalapril
Oxygen, acepromazine, and hydromorphone
Oxygen, furosemide, and nitroprusside
Oxygen, ampicillin, and enrofloxacin

A

From the radiograph, you should appreciate the cardiomegaly characterized by left ventricular enlargement, dorsally displacing the trachea on the lateral film, left atrial enlargement seen at the caudal heart base and compressing the mainstem bronchi, and widening of the cardiac silhouette suggestive of right-sided cardiomegaly. Left-sided failure is evident from the enlarged pulmonary veins and interstitial densities seen in the perihilar region. Right-sided failure is also suspicious given the wide cardiac silhouette and hepatomegaly with the gastric axis shifted caudally. Oxygen, furosemide, and nitrates (nitroglycerin or nitroprusside); remembering that nitroprusside provides both venodilation and arteriodilation, while nitroglycerin principally causes venodilation, are the primary treatments for acute congestive heart failure. This dog’s left-sided congestive heart failure is likely secondary to severe mitral valve degeneration and regurgitation. Sedation or antibiotics are not effective treatments for heart failure. Atenolol is a beta blocker and is useful for certain types of heart disease including hypertrophic cardiomyopathy in cats, but should not be given to patients in heart failure due to its negative intropic effects. Enalapril is an angiotensin converting enzyme inhibitor and should be given to this dog more chronically but will not help in this acute phase.

156
Q

Which of these compounds is an antidote for ethylene glycol toxicity?

4-Methylpyrazole Correct Answer
2-Pralidoxime
1,25 cholecalciferol
3-Methyl indole

A

The correct answer is 4-Methylpyrazole (4-MP). It is used to inhibit alcohol dehydrogenase and is considered the preferred treatment for treating ethylene glycol toxicoses in dogs. 4-MP does not cause hyperosmolality, metabolic acidosis, and CNS depression like ethanol treatment can. 4-MP is given to dogs IV over a 36-hour period. The initial dose is 20 mg/kg (slow IV over 15-30 minutes), then 15mg/kg (slow IV) at 12 and 24 hours, and then 5mg/kg is given at 36 hours. 4-MP is not effective in cats.

157
Q

A 7-year old German Shepherd dog presents for a second opinion of bilateral distal forelimb wounds. The owners note that they are not excessively pruritic but the patient will lick at the region. A biopsy had previously been performed with a microscopic description of chronic focal nodular pyogranulomatous dermatitis and destructive furunculosis without evidence of neoplasia or underlying etiology. In light of these findings, acral lick dermatitis is suspected. Which of the following is NOT a reasonable treatment strategy?

Ivermectin and prednisone therapy
Fluoxetine
Basket muzzle or E-collar
Combined systemic triamcinalone and clindamycin therapy

A

Acral lick dermatitis is considered a chronic and repetitive condition in which a dog continuously traumatizes a region as a result of obsessive licking. The licking is usually secondary to pruritis from allergies and will lead to infection. There could be multiple causes for this such as a neuropathy or underlying orthopedic disease; however, this behavior is considered to be obsessive-compulsive, and the patient may need to be evaluated for other anxiety related behaviors.

Since these lesions are not parasitic in origin the answer choice with Ivermectin is considered the poorest treatment option. All other choices may be used alone or in combination to help break the cycle and achieve healing of the lesion. Triamcinalone is a steroid. Clindamycin is an antibiotic. Fluoxetine (aka Prozac) is used to treat depression and obsessive-compulsive disorders. Basket muzzles and e-collars are commonly used as well to help stop the licking.

158
Q

You are treating an 8-year old mix breed terrier for ventricular tachycardia post splenectomy. You determine that you’ve given too much lidocaine to the patient. What clinical signs or laboratory findings are you most likely to see initially?

 Bradycardia and hypotension
 Diarrhea
 Methemoglobinemia
 Increased respiratory rate and difficulty breathing
 Muscle tremors and seizures
A

Early clinical signs associated with lidocaine toxicity include neurological symptoms such as seizures and tremors. Nausea and vomiting may occur, but is usually transient. Cardiovascular and respiratory depression can also occur, but usually later on in the course of clinical signs.

159
Q

A 6 year old Cocker Spaniel with a chronic history of ear infections presents to you circling to the left, with a left head tilt, and both rotary and horizontal nystagmus with the fast phase to the right. No other neurologic deficits are apparent except for a possible hearing deficit. It is difficult to perform a thorough otoscopic examination on the patient without sedation. The dog has been on multiple topical and systemic medications to treat otitis over the past few weeks, but the owner is not sure what they are. Which of the following drugs is a commonly implicated cause of peripheral vestibular disease?

 Metronidazole
 Miconazole
 Dimethyl sulfoxide (DMSO)
 Ticarcillin
 Chlorhexidine Correct Answer
 Enrofloxacin
A

The key here is to recognize that the question is asking about peripheral vestibular disease; otherwise, metronidazole would be a good choice as a cause of central vestibular disease. Chlorhexadine is a topical drug implicated in ototoxicity and peripheral vestibular disease. Another excellent choice would be aminoglycosides including neomycin, kanamycin, tobramycin, amikacin and gentamicin.

160
Q

Which of the following is the most likely diagnosis for cough in a 2-month old puppy?

Toxocara canis infection
Dirofilaria immitis infection
Trichuris vulpis infection
Toxascaris leonina infection

A

The correct answer is Toxocara canis infection. Toxocara canis is a canine roundworm that is passed along to puppies transplacentally. The roundworms migrate from the small intestines through the liver and lungs, which is what causes the animal to cough. Beyond 6 months of age, the larvae distribute to the somatic tissue, where their development is arrested. Dirofilaria immitis or heartworm disease takes at least 6 months before the infection is patent. It would take even longer for the infection to progress to congestive heart failure, causing the dog to cough. Trichuris vulpis, the canine whipworm, and Toxascaris leonina, another canine roundworm, do not migrate through the lungs.

161
Q

Which of the following statements about mammary tumors is true?

Development of mammary tumors in cats is not influenced by hormone levels through a cat’s life
Most mammary tumors in dogs occur in the 1st or 2nd glands
A 4 cm mammary carcinoma in a dog carries a better prognosis than a 2 cm mammary adenoma in a cat
A cat with a 3 cm mammary carcinoma has a good long term prognosis with surgery and chemotherapy
Dogs with malignant mammary tumors greater than 3 cm in size are considered to have a worse prognosis than those with tumors 2 cm in size

A

Generally, dogs with malignant mammary tumors greater than 3 cm are thought to have a worse prognosis than those with tumors smaller than 3 cm. The tumor size cut off in cats is 2 cm; cats with mammary tumors greater than 2 cm have a poorer prognosis than those with tumors smaller than 2 cm. Mammary tumors more frequently develop in the caudal glands (4th and 5th). Like in dogs, the risk for development of mammary tumors in cats is greatly reduced if they are spayed early in life; there is a strong hormonal influence associated with development of mammary tumors later in life.

162
Q

A 1-year-old female spayed Corgi presents for a seizure. At home, she vomited what appeared to be paint chips. Peripheral blood cytology and a complete blood count showed 20 nucleated red blood cells/100 white blood cells, basophilic stippling, and a neutrophilic leukocytosis (see image). She received a dose of diazepam, which has controlled her seizures. What is the treatment of choice?

Doxycycline
Ca-EDTA Correct Answer
N-acetylcysteine
D-penacillamine

A

This dog is showing signs of lead poisoning. Clinical signs are primarily gastrointestinal and neurologic. Animals are usually exposed in old buildings or areas of renovation where they have access to lead paint, old batteries, lead fishing weights, etc. Old food and water dishes can also be a source if lead paint was used. Lead blood levels can be measured; however, they do not necessarily correlate with severity of clinical signs. Toxic blood levels are greater than 0.4 ppm. Bloodwork can show an elevated number of nucleated RBCs without anemia. Basophilic stippling is a classic finding with lead toxicity but not specific. Ca-EDTA is the chelator used to treat lead poisoning. Succimer can also be given orally. D-penacillamine has also been used to chelate lead, copper, iron and mercury. It is used more commonly for copper toxicity. N-acetylcysteine is used for acetaminophen toxicity and hepatotoxicity. Basophilic stippling can sometimes be confused with erythrocytic parasites for which doxycycline may be the treatment of choice.

163
Q

Which of the following is a possible sequela to a CN VII (facial nerve) deficit?

Nystagmus
Strabismus of the eyes
Exposure keratitis
Loss of motor function in the muscles of mastication

A

The correct answer is exposure keratitis. CN VII is responsible for lacrimation. Deficits in lacrimation would cause exposure keratitis. Loss of motor function in the muscles of mastication would occur in CN V (mandibular branch) deficits. Strabismus of the eyes would be caused by deficits in CN III (oculomotor nerve), CN IV (trochlear nerve) or CN VI (abducent nerve). Nystagmus occurs with CNS disease or CN VIII deficits.

164
Q

A 4-week old intact male Gordon Setter presents for an acutely swollen face and lethargy. On physical examination you note pronounced submandibular lymphadenopathy, and occasional pustules on the face in addition to the swelling. Cytology does not reveal an infectious cause. Which of the following is an appropriate treatment?

Ketoconazole
Immunosuppressive doses of glucocorticoids
Cephalexin
Anti-inflammatory doses of glucocorticoids

A

This is a classic clinical presentation for juvenile cellulitis (puppy strangles). This disease is characterized by sterile granulomas and pustules that respond dramatically to glucocorticoids. The cause is thought to be an underlying immune dysfunction. Immunosuppressive glucocorticoid therapy should not be started until other infectious diseases have been ruled out. At a minimum a deep skin scrape, skin cytology, and trichogram should be performed to rule out Demodex, bacteria, and fungal infection.

165
Q

Can’t use with afib?

 Digoxin
 Isoproterenol
 Diltiazem
 Atenolol
 Procainamide
A

This ECG shows atrial fibrillation. Atenolol is a beta-blocker and will slow AV nodal conduction to decrease the ventricular response rate to atrial fibrillation. Procainamide is a class 1A anti-arrhythmic and can be used in attempts to convert atrial fibrillation to a normal sinus rhythm, though it is rarely effective. Diltiazem is a calcium channel blocker and can be used to slow AV nodal conduction and ventricular response rate. Digoxin will increase vagal tone to the AV node to slow conduction and decrease the heart rate. Isoproterenol is a non-specific beta receptor agonist, and will increase the ventricular rate in response to atrial fibrillation, thus is inappropriate in the management of this dysrhythmia.

166
Q

A 9 year old Greyhound presents to your clinic with an ulcerating mast cell tumor, located subcutaneously over the left scapula. You immediately schedule the dog for a wide surgical excision after no evidence of metastatic disease is identified on a complete blood count, serum chemistry, thoracic radiographs, and abdominal ultrasound. Your technician asks you if there are any drugs she should avoid using in this dog. You tell her the following drugs are contraindicated in this patient:

 Ivermectin and thiopental
 Atracurium and acepromazine 
 Propofol and morphine
 Acepromazine and ivermectin
 Morphine and thiopental
A

In this case, thiopental is contraindicated because the patient is a sighthound. Thiopental is an ultra-short acting barbiturate. Recovery depends on redistribution to tissues, including fat. Because sighthounds have very little fat, they have prolonged recoveries and greater complications with these drugs.

Morphine is contraindicated in this patient due to the ulcerating mast cell tumor. Morphine can cause histamine release and should be avoided in mast cell tumor patients.

167
Q

What adverse reaction are Dobermans predisposed to when using trimethoprim-sulfa?

Gastrointestinal ulcer formation
Hypersensitivity reaction
Vestibular disease
Pancreatitis

A

The correct answer is hypersensitivity reaction. Doberman Pinschers are more susceptible to hypersensitivity reactions from sulfa drugs as compared to other breeds. Type III reactions can result in immune complex disease which can cause arthritis, nephritis, and uveitis. Other possible side effects from this class of antibiotics that are not specific to the Doberman include KCS, hepatitis, and blood dyscrasias.

168
Q

An 8-month old female Newfoundland presents for further evaluation as a result of a recent onset of intermittent lameness in the right forelimb. On physical examination, there is pain on flexion and extension of the elbow joint. Radiographs of the elbow show a blunted medial coronoid process, sclerosis of the subchondral bone along the trochlear notch of the ulna and mild degenerative joint disease. What recommendations will you make to the owners?

Arthroscopy for evaluation and treatment of fragmented medial coronoid process Begin strict exercise restriction and anti-inflammatory therapy
Obtain joint cultures and start doxycycline immediately
Place a lateral splint and immobilize the limb for six weeks

A

The radiographic findings are consistent with a fragmented medial coronoid process. A CT scan may be recommended to help evaluate the joint prior to arthroscopy; however, this was not offered as a choice. During arthroscopy the surgeon will be able to carefully evaluate the joint for other components of elbow dysplasia such as OCD and elbow incongruency. In addition to evaluation, the surgeon will also be able to treat the condition by performing a sub-total coronoidectomy and removing and diseased cartilage if OCD is present.

169
Q

You are examining the eyes of a 4-year old German Shepherd. You see corneal melanosis and vascularization along the lateral aspect of the limbus. Both eyes show similar lesions. A Schirmer tear test is normal, and fluorescein dye is negative for stain uptake. What is the most likely diagnosis?

Chronic superficial keratitis
Trauma
Ocular lymphoma
Keratoconjunctivitis sicca

A

The answer is chronic superficial keratitis, sometimes referred to as pannus. It is a condition seen mainly in German Shepherd and shepherd crosses and is due to UV light exposure. It is believed that the UV light alters corneal proteins leading to an immune reaction. It is treated with topical steroids and cyclosporine, and animals often require lifelong therapy. KCS is not the correct choice because of the normal tear test. Lymphoma in the eye should cause changes in the uvea or retina but rarely the cornea.

170
Q

What is the end-point of warfarin therapy in an animal with a pulmonary thromboembolism?

A partial thromboplastin time of 1.5 to 2 times normal
A thrombin time of 1.5 to 2 times normal
A prothrombin time of 1.5 to 2 times normal
An activated clotting time of 1.5 to 2 times normal

A

The correct answer is a PT time of 1.5 to 2 times normal. PT is the best of these indexes to monitor warfarin therapy since, at these doses (usually 0.1-0.2 mg/kg), PTT and ACT will be unchanged. An even better method is to use the international normalization ratio (INR), which is a fancy way of taking into account how different PT reactions are run. The goal is to get an INR of 2.0 to 3.0.

171
Q

A 10-year old female spayed boxer presented for a mass on the left side of the neck. The mass was diagnosed as a mast cell tumor based on fine needle aspirate. Screening with thoracic radiographs, abdominal ultrasound and bloodwork was unremarkable. The owners elected for surgical removal and clean 3 cm margins were obtained in all directions. The mass was submitted for histopathology and was categorized as a grade 3 (or high grade) tumor. What is your recommendation to the owner?

Start a tyrosine kinase inhibitor chemotherapy drug.
Radiation therapy for the surgical site and prescapular lymph node; abdominal ultrasounds every 3-4 months.
Start a doxorubicin based chemotherapy protocol.
Monitor the surgical site for recurrence; abdominal ultrasounds every 3-4 months.

A

Under the Patnaik grading system, grade 3 tumors are highly metastatic. Mast cell tumors are highly responsive to radiation therapy, but with clean excision and wide surgical margins, local recurrence is unlikely. Metastasis is the primary concern and a wait and monitor approach is not advised. Even with no evidence of metastasis at the time of surgery, chemotherapy should be initiated as soon as possible. Common protocols include single agent or alternating vinblastine and lomustine. Tyrosine kinase inhibitors, such as toceranib (Palladia) and masitinib (Kinavet, which is no longer available), target c-kit, a stem cell factor receptor involved in mast cell proliferation and differentiation. Palladia and Kinavet are FDA approved for treatment of high-grade canine cutaneous mast cell tumors.

172
Q

A 10-year old female spayed greyhound presents with severe dental and periodontal disease (see image). You perform pre-anesthetic bloodwork, which is within normal limits and schedule the dog for a dental cleaning next week. Which of the following can be recommended for short-term use to treat periodontal disease in dogs?

Daily chlorhexidine rinses
Raw hide chews
Enrofloxacin
Daily periodontal probing

A

Chlorhexidine rinses can be used short-term, such as prior to or immediately after any dental procedure. Daily usage kills off normal bacterial flora in the mouth and will stain teeth over time. Daily chlorhexidine use is sometimes advocated as a long-term treatment of severe periodontal disease, but alternative therapies should be recommended if possible. Raw hide chews, daily teeth brushing, and chew toys all help to reduce plaque and calculus when used regularly.

Enrofloxacin is a not a very good choice for oral infections since it is ineffective against anaerobic bacteria. Beta-lactam antibiotics (especially Clavamox) or clindamycin are better choices for oral infections because of their activity against anaerobes.

173
Q

You get called to a farm that is having trouble with mid-term abortion in their cows. As you are driving up to the farm you see one of the farm dogs and you notice that the dog appears to be suffering from diffuse muscle atrophy and has trouble ambulating in the hindlimbs. You immediately make an association between the dog and the abortions. What is your primary differential?

 Neospora caninum 
 Epizootic bovine abortion
 Toxoplasma gondii
 Chlamydophila felis
 Brucella bovis
A

The correct answer is Neospora caninum, a protozoan parasite. The definitive host is the dog (or coyote or other canidae) which acquires the infection by eating infected meat and spreads it to cows by shedding oocysts in the pasture or feed. Although clinical signs are mainly seen as abortions and abnormal calves in herbivores, dogs may suffer from neurologic and muscular abnormalities. Infection to other dogs is usually as a result of transplacental spread. IFA is usually performed to make the diagnosis.

174
Q

Which of the following is the treatment of choice for pulmonic stenosis in the dog?

Surgical valvulotomy
Valvulotomy by catheterization device
Balloon valvuloplasty
Patch grafting of the outflow tract

A

The correct answer is balloon valvuloplasty. This procedure is performed by inserting a catheter into either a jugular vein or a femoral vein and advancing it into the region of stenosis at the pulmonic outflow tract. Once in place, a balloon surrounding the catheter is inflated to stretch the stenotic region open. Surgical valvulotomy or patch grafting is the treatment used for severe cases that cannot be balloon-dilated.

175
Q

Approved tx for metastatic melanoma?

 Mitoxantrone
 ONCEPT vaccine Correct Answer
 Palladia
 Vincristine
 Doxorubicin
A

The image shows a lymph node with metastatic melanoma (the large cell with black melanin pigment). The ONCEPT vaccine, also commonly referred to as the “melanoma vaccine” was given a conditional product license in 2007 and was USDA approved in 2010 as a therapeutic vaccine for cancer treatment.

Palladia is a receptor tyrosine kinase inhibitor approved for use in mast cell tumors in dogs. Doxorubicin, vincristine and mitoxantrone are human chemotherapy drugs commonly used off-label in veterinary oncology but not routinely for melanoma.

176
Q

What is the hormonal profile of a female dog in estrus?

Falling estrogen, falling progesterone
Falling estrogen, rising progesterone
Rising estrogen, rising progesterone
Rising estrogen, falling progesterone

A

The correct answer is falling estrogen, rising progesterone. This is what stimulates the LH surge and leads to ovulation and the behavioral change in estrus where a female will stand to be bred.

177
Q

On the morning of a beautiful summer day, a five-year old, male German Pointer played unattended in the backyard garden. During the following 24 hours the right side of the dog’s face became swollen as seen in the image below. The dog developed urticaria and became lethargic, unable to walk and vomited several times. Which of the following are the best treatment options for the likely condition?

Furosemide, oxygen, and nitroglycerin
Drain the swelling, administer antibiotics
Diazepam, methocarbamol, and non-steroidal anti-inflammatories
Induce vomiting, administer activated charcoal, intravenous fluids
Antihistamines, corticosteroids, and epinephrine

A

The dog has angioedema +/- anaphylaxis, likely caused by an insect bite or sting. A snake bite should also be ruled out by trying to visualize the bite or questioning the owners. This is a type I hypersensitivity reaction and is treated by removing the offending agent if possible and providing supportive care with anthistamines, corticosteroids, and epinephrine as needed.

178
Q

A dark red vaginal discharge in a bitch 3 weeks after whelping indicates what process?

 Subinvolution of placental sites
 Pyometra
 Retained placentas
 Normal response 
 Metritis
A

The correct answer is normal response. Dark red vaginal discharge occurs normally for up to 6 weeks after whelping. Subinvolution can cause fresh bleeding for 12-15 weeks post-partum. Metritis is usually accompanied by systemic signs and may have abnormal vaginal discharge but not usually hemorrhage. Retained placentas typically do not occur in bitches. Pyometras occur during the luteal phase when progesterone levels are high, up to 9 weeks after estrus, and would not occur post-whelping.

179
Q

A 1-year-old male castrated Basset Hound presents for epistaxis from both nostrils, a small amount from blood from the right and a larger amount from the left. There is no history of trauma but he does go for frequent walks, and as a typical Basset, usually has his nose to the ground. His heart and lungs sound normal. You note a few ecchymotic hemorrhages on the thin skin under his abdominal region. You discuss the various causes of this issue with the owner and perform some labwork, including a coagulation panel, and the important values to note are as follows:

PCV 34%
Reticulocytes 110K
Platelets 180K/uL (143-448 K/uL)
Prothrombin Time (PT) 6.9s (6.3-13.3 seconds)
Partial Thromboplastin Time (PTT) 10.9s (10.6-16.8 seconds)

Canine thrombopathia
Nasal foreign body Your Answer
Rodenticide toxicity
Immune-mediated thrombocytopenia

A

The correct answer is canine thrombopathia. The disease is an autosomal recessive trait seen in Basset Hounds where platelets fail to aggregate and secrete their granules in response to normal stimuli. These dogs are at an increased risk for bleeding spontaneously, and an injury or surgical procedure could cause excessive hemorrhage. Platelet numbers and coagulation parameters are normal in this condition. It isn’t an issue with platelet number, but of function.

Rodenticide would cause elevated clotting times.

Immune-mediated thrombocytopenia would cause a low platelet count.

A nasal foreign body would be less likely to cause a bilateral bleed and would not attribute to the ecchymotic hemorrhages noted on the abdominal skin.

180
Q

The mean circulating life span of a white blood cell in a dog is approximately _______.

120 minutes
48 hours
5-9 hours
5-9 days

A

The correct answer is 5-9 hours.

Platelets have a circulating half life of 5-7 days and red blood cells have an approximate life span of 110 days in dogs

Note: The approximate life span of red blood cells in other species (in days) is as follows: cats-70, cow-160, horse-145, pig-86, sheep- 150

181
Q

Which of these is least likely a cause for anterior uveitis as shown in this dog with aqueous flare?

 Trauma
 Coccidioides immitis infection
 Systemic Lupus Erythematosis 
 Hypertension
 Lymphoma
A

The correct answer is systemic lupus erythematosis. SLE causes many signs, but anterior uveitis is not considered one of them. The causes of uveitis include infection (bacterial, viral, fungal, rickettsial), immune mediated (lens-induced uveitis), neoplasia, hypertension, and trauma. Over 50% of the time, the cause is not found and the uveitis is termed idiopathic.

182
Q

Which of the following is an osmotic diuretic?

 Mannitol 
 Acetazolamide
 Spironolactone 
 Enalapril
 Furosemide
A

The correct answer is mannitol. Mannitol is filtered at the glomerulus and osmotically pulls water into the tubules. Furosemide is a loop diuretic, acting at the loop of Henle. Enalapril is an angiotensin-converting enzyme (ACE) inhibitor. Spironolactone is an aldosterone antagonist. Acetazolamide is a carbonic anhydrase inhibitor.

183
Q

A 1-year-old female Golden Retriever “Stella” presents with bilateral exophthalmos that has taken place over the past 2 days.

T= 102.2, HR 140, RR 30. Lab work is unremarkable. She does not seem painful, and no swelling around the eyes is evident. Pupillary light response is normal in both eyes and there is minimal resistance to retropulsion of the globes. The masticatory muscles do not appear to be affected and there is no difficulty opening the mouth. Stella is apparently healthy other than the exophthalmos. What condition do you suspect?

Lymphoma
Orbital neoplasia
Retrobulbar abscess
Extraocular polymyositis

A

The answer is extraocular polymyositis. Extraocular polymyositis is a condition seen primarily in young Goldens and they present with the signs above. It is kind of similar to masticatory muscle myositis in that it occurs due to an autoimmune reaction against muscle antigens. It is treated with prednisone +/- azathioprine. The condition can be differentiated from these other conditions because orbital neoplasia and retrobulbar abscesses are usually unilateral. Abscesses are usually accompanied by more systemic signs. Orbital neoplasia is usually more slowly progressive and seen in older animals. Lymphoma could present in a similar fashion but is less likely in such a young dog with no other signs.

184
Q

You are suspicious that the dog may have immune-mediated polyarthritis (IMPA). Which of the following are all known potential inciting causes or predisposing factors for this condition?

Penetrating joint trauma, gastric dilatation-volvulus, ingestion of onions
Systemic lupus erythematosus, sulfonamide exposure, mammary adenocarcinoma
Hemangiosarcoma, fungal osteomyelitis, cruciate ligament disease
Synovial cell sarcoma, renal insufficiency, hyperthyroidism
Hypothyroidism, anal sac carcinoma, leptospirosis vaccination

A

IMPA is often classified as being erosive or nonerosive. This case is an example of the nonerosive form based on the lack of radiographic evidence of cartilage or subchondral bone destruction. The erosive form is rare and is thought to account for <1% of the cases of IMPA.

IMPA can be associated with a variety of systemic diseases or precipitating factors including systemic infectious, inflammatory or neoplastic disease or reactions to drugs or vaccines. Systemic lupus erythematosus is a progressive multiorgan autoimmune disease which frequently manifests with polyarthritis and may also cause concurrent hemolytic anemia, thrombocytopenia, glomerulonephritis, skin lesions, and lymphadenopathy. Drug induced IMPA has been associated with a variety of medications including sulfonamides, lincomycin, erythromycin, cephalosporins, phenobarbital, and penicillins. Vaccine associated polyarthritis has been suggested but association has been difficult to prove. Some breeds of dogs have particular predispositions to IMPA including Akitas (in association with meningitis) and Shar-Peis (in association with “Shar-Pei fever”). IMPA can also be associated with chronic infectious or inflammatory diseases including pyoderma, urinary tract infections, pneumonia, endocarditis, mastitis, heartworm, fungal infection, and severe periodontal disease. Chronic inflammatory bowel disease, intestinal overgrowth, and ulcerative colitis have been associated with IMPA. Distant neoplasia has also been associated with IMPA including squamous cell carcinoma, mammary adenocarcinoma, leiomyoma, heart based tumors, and seminoma.

The clinical signs associated with IMPA are anorexia, weight loss, fever, lethargy, and lymphadenopathy. Up to 25% of dogs present with only nonspecific signs of systemic illness and without apparent gait abnormality or joint effusion. It is an important consideration for dogs with fever of unknown origin and may be the cause up to 20% of the time. IMPA is diagnosed by synovial fluid analysis although additional baseline diagnostics are indicated to screen for potential systemic or infectious causes. Normal synovial fluid is clear and viscous with <2.5 g/dl protein and <3,000 cells/ul with predominantly mononuclear cells. Joint fluid in IMPA may be thin, turbid, and increased in volume with higher amounts of protein and cells, often primarily nondegenerate neutrophils.

Treatment of idiopathic IMPA centers on immunosuppressive therapy, often starting with prednisone and sometimes including an additional immunosuppressive drug such as azathioprine or cyclophosphamide. About 80% of dogs will respond to immunosuppressive doses of prednisone; however, half of dogs require long-term or additional drug therapy to maintain remission.

185
Q

Two 9-year old female spayed Labrador retrievers presented for their next chemotherapy treatments. Both were diagnosed with multicentric lymphoma and have been receiving the CHOP protocol. One received doxorubicin (Adriamycin) and one received vincristine (Oncovin) today. Your technician informs you that both unfortunately had extravasations occur and wants to know what she should do. Which drug is a worse vesicant and what should be done to the injection site?

Doxorubicin. Cold compress to contain the spread of drug
Vincristine. Warm compress to disperse the drug
Doxorubicin. Warm compress to disperse the drug
Vincristine. Cold compress to contain the spread of drug

A

Intravenous chemotherapy drugs can cause severe tissue necrosis (doxorubicin) or irritation (vincristine) if extravasation occurs. Treatment for both should begin immediately. In both cases, the catheter should be left in place and as much of the drug should be aspirated as possible. Treatment for doxorubicin extravasation involves cold compressing the site to promote vasoconstriction, however perivascular necrosis may still occur and may progress days to weeks later. In severe cases involving doxorubicin, debridement or limb amputation may be needed.

Vincristine extravasation should be treated with warm compresses to disperse the drug and enhance systemic absorption.

Extravasation should be prevented through patient restraint training, use of catheters that have been placed on the first stick, and careful monitoring during administration. The peripheral veins should be avoided for blood draws if possible on all patients receiving chemotherapy.

186
Q

A 5-year-old Doberman presents for chronic pruritus and recurrent bacterial skin infections. The patient has been on a 4-week course of Cephalexin and continues to get new skin lesions despite this treatment. You suspect methicillin resistance and recommend a skin culture. The owner declines a culture and would like you to select another antibiotic empirically. Which antibiotic would be the best choice considering side effects and efficacy?

 Trimethoprim-sulfa
 Clindamycin 
 Cefpodoxime (Simplicef)
 Cefovecin (Convenia)
 Chloramphenicol
A

Since you are suspicious of methicillin resistance, using any cephalosporins such as Convenia or Simplicef would not be useful. Trimethoprim-sulfa will probably be effective; however, this antibiotic should be avoided in Dobermans as they are prone to side effects such as anemia, thrombocytopenia, arthritis, and renal toxicity. These side effects are due to a type-three hypersensitivity. Chloramphenicol may also be effective against the infection, but clindamycin would be a better choice, as it has fewer side effects. Chloramphenicol can cause peripheral neuropathy, gastrointestinal upset, bone marrow suppression, and hepatotoxicity. Additionally, chloramphenicol can also cause aplastic anemia in humans. Clindamycin can also cause gastrointestinal upset but seems to be better tolerated than chloramphenicol.

187
Q

A 5-month old Scottish terrier presents to you with signs of head pressing and seizures. It was previously diagnosed with a portosystemic shunt, and its abdominal radiographs show a small liver (see radiographs). Which of these should NOT be used to manage hepatic encephalopathy in dogs with a liver shunt?

Lactulose
Substituting dairy and vegetable protein instead of feeding meat proteins
Oral neomycin
Low carbohydrate and fat, high protein diet

A

The answer is low carbohydrate and fat, high protein diet. Animals with hepatic encephalopathy require protein reduction in their diets. Lactulose is given because it decreases colonic pH, trapping ammonia, and decreases transit time through the gut, leading to decreased ammonia uptake because ammonia contributes to hepatic encephalopathy. Oral neomycin is used to dampen urease-producing microflora of the gut and consequently, decrease ammonia production in the gut. It has also been shown that vegetable and dairy proteins are better tolerated than meat proteins.

On the radiograph, note the liver tucked well under the rib cage and the corresponding cranial shift of the gastric axis consistent with microhepatica.

188
Q

Which of the following statements is true about central venous pressure?

The measurement is independent of cardiac output
It is a direct measure of blood pressure in the cranial vena cava
It is a direct measure of preload
The measurement is independent of venous vessel tone
It is an estimation of afterload

A

The correct answer is it is a direct measure of blood pressure in the cranial vena cava. Venous blood volume, venous vessel tone, and cardiac output determine the CVP. CVP is an estimate of preload, not a direct measurement. Cardiac output is one of the factors along with venous tone and venous blood volume in obtaining CVP. Afterload is the force against which the myocardium must contract.

189
Q

Zoonotic diseases commonly carried by raccoons include rabies and ____________.

Baylisascaris Correct Answer
Toxocara
Echinococcus
Trichinella

A

The correct answer is Baylisascaris. Baylisascaris procyonis is an ascarid parasite of raccoons that causes mild signs in raccoons but can undergo aberrant migration in humans and cause fatal central nervous system signs.

190
Q

Which of these breeds of dogs has a congenital ocular anomaly characterized by choroidal hypoplasia, manifested by varying degrees of visual dysfunction with signs of large bizarre choroidal vessels visible on fundic exam as depicted here?

 Boston Terrier
 German Shepherd Dog
 Collie Correct Answer
 West Highland White Terrier
 Toy Poodle
A

The correct answer is collie. Collie Eye Anomaly is seen in up to 80% of the breed. Some are completely blind and others show no visual deficits. Other signs include optic disc coloboma, retinal hemorrhage and retinal separation.

191
Q

Which of these fungal pathogens grow in pigeon feces and has the potential of being a serious zoonotic problem?

 Sporothrix
 Coccidioides
 Aspergillus
 Histoplasma
 Cryptococcus
A

The correct answer is Cryptococcus. Cryptococcus neoformans grows exceptionally well in pigeon feces. Blastomyces is also found in bird feces. Histoplasma can grow in bird feces but is not as much of a zoonotic concern.

192
Q

What should be told to owners of dogs that are infected with Echinococcus granulosus?

a. The parasite is transmitted by the ingestion of fleas, so the dog should be treated for fleas
b. The parasite causes hydatid cyst disease in humans, which can be fatal
c. The dog acquired the infection by ingesting a rat or bird
d. The parasite is highly pathogenic and causes hydatid cyst disease in dogs, but is not a zoonotic threat to humans

A

The correct answer is the parasite causes hydatid cyst disease in humans, which can be fatal. Echinococcus granulosus is the hydatid tapeworm which is non-pathogenic in dogs but is highly pathogenic to fatal in humans and other intermediate hosts in which a hydatid cyst forms. The infection is acquired in dogs by eating raw sheep meat or viscera infected with the parasite. All dogs suspected of being infected should be treated with praziquantel.

193
Q

A couple comes in frantic to your hospital with their two Labrador retrievers. Both dogs are unable to walk and need to be carried into the hospital. The owners had been camping with the dogs over the weekend and report that one of the dogs started to show signs of weakness in the hind end near the end of the trip. They assumed he was tired from running around as their other dog seemed more quiet on the way home as well. This morning when they woke up neither dog could stand and one of the dogs had trouble eating breakfast. Both dogs are up to date on vaccinations and preventatives.
The dogs are non-painful, tetraplegic, hypo-reflexive in all four limbs, superficial and deep pain intact, and their tails continue to wag. One of the dogs has decreased tongue tone. You do not find any external lesions. Initial complete blood count, chemistry panel and urinalysis are normal.
Of the following options, what is the most likely diagnosis for both dogs?

 Botulism 
 Coonhound paralysis
 Myasthenia gravis
 Tick bite paralysis
 Intervertebral disc disease
A

When two patients are affected with the same symptoms your first thoughts should be something contagious, a toxin, or an environmental factor. All of the options are reasonable explanations for worsening paralysis; however, the history of camping and two patients being affected make something infectious high on your list of differentials. Intervertebral disc disease would be highly unusual to affect two large-breed dogs at the same time, and pain is often initially noticed on spinal palpation. Coonhound paralysis or idiopathic polyradiculoneuritis is often preceded by raccoon bite, systemic illness or vaccination, and no external wounds were seen on either patient. Tick bite paralysis resolves when the tick is removed. Myasthenia gravis can show similar signs (especially the difficulty with eating and swallowing), but it tends to show some improvement after rest and would be unlikely to spontaneously occur in two patients at the same time. Botulism is contracted from eating contaminated food containing the Clostridium botulinum type C neurotoxin. It can be found in carrion and spoiled food. Given that the dogs were out camping, they likely ate contaminated food when the owners weren’t looking and therefore became ill at the same time. The difference in severity between patients is likely related to the quantity of toxin ingested.

194
Q

Demodicosis often coincides with bacterial infections. On a recheck examination of an 8-month old pit bull with demodicosis you find swelling and furunculosis over the lateral thighs that is draining serosanguinous fluid. Cytology of the fluid reveals 4+ cocci, and you diagnose a deep bacterial pyoderma. You culture the lesion and start Clindamycin based on the culture results. The dog returns in four weeks for another recheck examination. No bacteria are found on cytology, and clinically the dog looks excellent. You repeat a skin scrape and find Demodex mites. You inform the owner to continue Ivermectin. What should you do regarding continuation of antibiotics?

Continue the antibiotics for another 2 weeks.
Discontinue the antibiotics, since cytology was negative and the clinical signs have resolved
Continue the antibiotics until the demodex has resolved
Perform another culture and if positive keep treating; if negative discontinue antibiotics

A

Deep pyodermas involve tissues deeper than the epidermis including the dermis and even subcutis. Most often deep infections have resulted from an uncontrolled superficial infection. Deep lesions usually heal on the surface before the deeper infection is resolved; making clinical cure difficult to assess. Therefore, antibiotic treatment should be continued for 7-21 days after the tissues return to normal. In general, deep infections can require 6-8 weeks of antibiotic treatment and even 12 weeks for severe cases. Superficial infections require 3-4 weeks of antibiotic therapy or one week past clinical resolution.
Culture results will often be positive in normal dogs since normal dogs will have bacteria on their skin surface and in hair follicles. It is better to manage based on clinical signs. Cytology can miss deeper infections and it is better to extend antibiotic treatment even if cytology is negative.
It is not necessary to continue the antibiotics until resolution of demodicosis. Keep in mind that many Demodex patients will require several months of treatment. It is better to prevent recurrence of bacterial infections with topical therapy such as benzoyl peroxide shampoos.

195
Q

A 3 year old female spayed mixed breed dog presents for lethargy and back pain. Survey radiographs of the spine show vertebral endplate lysis at C3-C4, C4-C5, and L6-L7. What is your next step?

Look for an occult infection somewhere else in the dog and perform blood cultures
Prescribe NSAIDs and cage rest at home
Take the dog to surgery to stabilize the spine
Perform a bone scan

A

Look for an occult infection somewhere else in the dog and perform blood cultures. The dog has discospondylitis, which is usually caused by hematogenous spread of bacteria from a distant infected site. Radiography showing vertebral endplate lysis is definitive for a diagnosis, but blood cultures and sensitivities should be run to determine what type of antibiotics should be used. Surgery is usually indicated if neurologic deficits are severe. Bone scans are usually only used when radiographic changes are not yet visible.

196
Q

What is the correct post-exposure prophylaxis protocol after being exposed to rabies for an unvaccinated human?

Five injections of approved rabies vaccine
One injection of human rabies immune globulin followed by 2 injections of approved rabies vaccine
Injection of human rabies immune globulin followed by 4 injections of approved rabies vaccine
Two injections of approved rabies vaccine
Three injections of human rabies immune globulin followed by 2 injections of approved rabies vaccine

A

The correct answer is injection of human rabies immune globulin followed by 4-5 injections of approved rabies vaccine. If you are not vaccinated, then you will need an injection of human rabies immune globulin immediately after exposure followed by 4-5 injections of an approved rabies vaccine IM over 2-4 weeks. If you have been vaccinated, all that is needed are two injections of an approved rabies vaccine 3 days apart. PEP is not required for accidental injection of animal rabies vaccine into a human.

197
Q

A 2-year old large mixed breed dog presented 2 weeks ago during your emergency rotation with a ventro-caudal hip luxation after jumping out of the back of a truck. You manually reduced the hip and applied hobbles. The dog seemed to do well, but now the hip has luxated again as confirmed with radiographs and the dog has been lame for the last 2 days. There are no fractures or bone pathology noted. At this time, a closed reduction will not be a good option due to the chronic damage to the joint capsule. Which procedure would be recommended?

Femoral head ostectomy
Toggle pin fixation
Amputation
Total hip replacement

A

Since this hip has luxated again and it has likely been going on for a couple of days, an open reduction surgery would now be recommended. Since the hip bones are otherwise healthy, a toggle pin fixation would be the best option for this dog.

If there were a fracture or severe osteoarthritis present, a total hip replacement or FHO would be other options to consider. Amputation would not be an option for this condition.

198
Q

A 2-year old male castrated Bull Mastiff with a history of travel in the west coast presents to your clinic for a chronic cough and a recent development of lameness of his right hind limb. On physical exam you notice a draining lesion over the lame region of the right hind limb. You perform chest radiographs and see a diffuse bronchointerstitial pattern which is nodular in some regions. You also identify hilar lymphadenopathy. You suspect that you know what you are dealing with and perform a broncho-alveolar lavage for cytology. Just as you suspected, you see spherules. What is your diagnosis?

 Blastomycosis
 Histoplasmosis
 Aspergillosis
 Coccidioidomycosis 
 Cryptococcus
A

The correct answer is Coccidioidomycosis. The travel history and clinical signs are consistent with this answer. Additionally, finding spherules on cytology is pathognomonic for Coccidioidomycosis. Prolonged antifungal treatment will be necessary. Fluconazole is the treatment of choice. Ketoconazole and itraconazole are good choices. With blastomycosis you see broad based budding of the yeast. With Cryptococcus neoformans you will see narrow-based budding.

199
Q

Which of the following is not true about doxycycline used in dogs?

It can cause discoloration of teeth and thin enamel in puppies
It can cause gastrointestinal upset
It can cause esophageal stricture formation if the drug remains in the esophagus for an extended time period
It can cause cartilage abnormalities in growing puppies

A

The correct answer is it can cause cartilage abnormalities in growing puppies. Fluoroquinolone antibiotics such as enrofloxacin can cause cartilage formation abnormalities in growing animals. Doxycycline is a tetracycline antibiotic that can cause esophageal stricture formation if it remains in the esophagus for an extended period of time. A small amount of food or water is usually given after the pill to ensure the tablet is not lodged in the esophagus. It can also cause teeth abnormalities in growing animals and often causes GI upset.

200
Q

A 7 month old male Labrador Retriever presents for lethargy, a distended abdomen, and exercise intolerance. Your physical exam reveals a 4/6 systolic murmur heard best at the right mid thorax. Thoracic radiographs show severe cardiomegaly with marked right atrial enlargement and right ventricular enlargement. The caudal vena cava is enlarged, and part of the abdomen in the film shows loss of serosal detail and hepatomegaly. What is your most likely diagnosis?

Tricuspid dysplasia
Hepatocellular carcinoma
Pulmonic stenosis
Hemangiosarcoma

A

The correct answer is tricuspid dysplasia. Tricuspid dysplasia is similar to mitral valve dysplasia in the irregular valve leaflets, chordae tendinae, or papillary muscles of the valve. Labrador Retrievers, German Shepherd Dogs, and other large breed males are predisposed to the disease. Signs of right heart failure, such as ascites and hepatomegaly occur in advanced disease. The prognosis for these animals is guarded to poor depending on the severity of the regurgitation. Pulmonic stenosis causes a pressure overload of the right heart and causes concentric hypertrophy, which may or may not be detectable by radiography. The murmur with PS is heard better at the left heart base. Hemangiosarcoma and hepatocellular carcinoma are less likely in a 7 month old puppy.

201
Q

Which value is not usually increased in canine hyperadrenocorticism?

SAP (serum alkaline phosphatase)
Blood urea nitrogen
ALT (alanine amino transferase)
Serum cholesterol

A

The correct answer is blood urea nitrogen (BUN). BUN is usually low in patients with hyperadrenocorticism secondary to the increased diuresis that occurs with elevated cortisol levels. ALT increases due to swelling and death of some hepatocytes. SAP increases due to a steroid induced isoenzyme of SAP from the liver. Mild cholestasis due to swelling of hepatocytes also contributes to a minor part of the increase in SAP. Cholesterol is elevated in up to 90% of hyperadrenocorticism cases.

202
Q

Which of these is not a sign of ongoing or acute chorioretinitis in dogs?

Poorly defined gray spots throughout the fundus
Retinal separation
Retinal hemorrhage
Multifocal areas of tapetal hyperreflectivity

A

The correct answer is multifocal areas of tapetal hyperreflectivity. Poorly defined gray spots indicate areas of cellular infiltration during acute chorioretinitis. Retinal separation and hemorrhage are also commonly seen in acute cases. Hyperreflectivity is a sign of previous damage or chronic chorioretinitis.

203
Q

Which of the following statements is true regarding a 6-year old Cocker Spaniel that has polyuria, polydipsia, and blood glucose consistently around 160 mg/dl (normal 76-119 mg/dl)?

The polyuria and polydipsia are not likely caused by diabetes mellitus. Further diagnostic tests should be pursued to look for the cause of polyuria and polydipsia.
A urinalysis should be run and will likely show large amounts of glucose and ketones in the urine.
The dog has diabetes mellitus. The polyuria is caused by osmotic diuresis of glucose spilling into the urine, and the polydipsia is secondary to the polyuria.
The dog has diabetes mellitus. The polyuria is caused by the kidneys being refractory to antidiuretic hormone. The polydipsia is secondary to the polyuria.

A

The correct answer is that PU and PD are not likely caused by diabetes mellitus. In a dog with normal kidneys, glucose does not begin to spill into the urine until the blood glucose is around 180 mg/dl. In a cat, glucose does not begin to spill into the urine until the blood glucose reaches between 200-280 mg/dl. Another cause for the PU, PD should be sought in this patient. When the blood glucose is high enough to spill into the urine, the polyuria seen is caused by an osmotic diuresis resulting from the osmotic pull of the glucose in the urine. The kidneys are not refractory to antidiuretic hormone with DM.

204
Q

A 10 year old female spayed Labrador Retriever has presented for difficulty getting up in the hind and apparent back pain for the last 3-4 weeks. Upon questioning the owner, you are told that her appetite is diminished but she is still eating. An orthopedic exam finds no pain or discomfort elicited on manipulation of the hips or stifles. A neurologic exam identifies substantial pain in the lumbosacral region; however, conscious proprioception is intact, and patellar reflexes are normal.

Radiographs of the lumbosacral region identify a lytic lesion at the L7-S1 endplates as well as surrounding bony proliferation. Which of the following diagnostic tests is likely to provide the most helpful additional information given your findings?

 Urine culture 
 Chest radiographs
 Abdominal ultrasound
 Stifle arthrocentesis
 Blood draw for creatine kinase levels
A

The diagnosis you should have in mind is discospondylitis. Be sure not to confuse this with spondylosis, which is typically not clinically significant and can be expected in most older dogs. The radiographic description is relatively specific for this condition. Neoplasia in the spine should be lower on your differential list because it typically does not cross joints. Disc herniation cannot be ruled out, and advanced imaging would be necessary to know for certain, but given the exam and radiographic findings, discospondylitis should be your top differential.

Discospondylitis is usually bacterial in origin, with Staphylococcus being the most common organism involved. Other organisms identified include Brucella canis, E. coli, Pasteurella, Aspergillus, and Streptococcus.

In an attempt to identify the causative agent, urine and blood cultures should be considered. The other answer choices are not as high yield in identifying the specific bacterial cause or underlying etiology.

205
Q

A previously healthy lactating dog 2 weeks after whelping shows signs of restlessness, whining, fever, muscle spasms and convulsions. What is the most likely cause?

Hypercalcemia
Dystocia
Pelvic fracture
Eclampsia

A

The correct answer is eclampsia. This should be suspected in a nursing dog with neurologic signs, pain or stiffness. It is due to reduction in serum-ionized calcium secondary to the demands of producing large amounts of milk. Treatment is with calcium supplementation.

206
Q

A 13-year-old, male neutered Labrador retriever patient presents for a quality of life consult. The owner is concerned that his dog has progressively become more painful in the hips. He was diagnosed with hip dysplasia at a young age and the owner elected to treat him conservatively by providing low impact exercise, maintaining a lean body weight, and administering non-steroidal anti-inflammatories as necessary for pain. There are several nutraceuticals available that have been touted as having some benefit in patients with osteoarthritis. Which of the following will result in an increased production of less inflammatory eicosanoids?

Carnitine
Chondroitin
Omega-3 fatty acids Correct Answer
Glucosamine

A

Consuming a high concentration of omega-3 fatty acids (eicosapentaenoic acid and docosahexaenoic acid) has been shown to result in the preferential use of these compounds to form eicosanoids. Eicosanoids derive from either omega-3 or omega-6 fatty acids. Omega-6 eicosanoids are generally pro-inflammatory, while omega-3 eicosanoids are less inflammatory. Omega-3 fatty acid supplementation results in a decrease in pro-inflammatory omega-6 eicosanoids; providing potential benefit to patients with osteoarthritis.

Carnitine is thought to potentially aid in weight loss as it is involved in fat metabolism, but there are limited studies demonstrating significant clinical benefit to supplementation at this time.

Glucosamine is a precursor of glycosaminoglycans which is a major constituent of the joint and thought to be of great importance for joint health. Chondroitin is also an important constituent of cartilage and helps provide resistance to compressive force. It is commonly administered in conjunction with glucosamine. The amount of clinical benefit of glucosamine and chondroitin supplementation is not currently well established.

207
Q

A 7-year old, male, castrated shepherd mix presents for polyuria and polydipsia. The physical exam reveals a body condition score of 4/9 and 5% dehydration. The urine specific gravity is 1.006. The urinalysis, chemistry panel, and CBC are otherwise unremarkable. Which of the following is the most likely diagnosis?

Chronic renal failure
Diabetes mellitus
Central diabetes insipidus
Fanconi syndrome

A

The correct answer is central diabetes insipidus.

Diabetes mellitus is ruled out with a normal blood glucose on the chemistry panel and lack of glucose in the urine on the urinalysis. Chronic renal failure is ruled out because there is no azotemia on the chemistry panel. Fanconi syndrome usually occurs in Basenjis and is ruled out because there is no glucose in the urine.

208
Q

One of your colleagues has asked you to be the anesthetist for a lung lobectomy in an 8 year old Pug with a lung lobe torsion. He stresses that there will be times in the procedure where it is absolutely critical that the dog stay completely still, including no respiratory motion. Which of the following drugs could be used to do this while maintaining the dog under a reasonable surgical plane of anesthesia?

Propofol constant rate infusion
Sevoflurane gas and injectable edrophonium
Sevoflurane gas and a fentanyl infusion
Isoflurane gas and a lidocaine infusion
Isoflurane gas and injectable succinylcholine

A

To control all respiratory motion without pushing a dog to an excessive and dangerous plane of anesthesia, a neuromuscular blocking agent would be necessary. You would never use one of these alone for a surgical procedure as they do not provide analgesia or reduce consciousness, which is why an inhalant gas anesthetic is also used to maintain anesthesia in this case.

Options include succinylcholine which is a depolarizing neuromuscular blocker or one of the nondepolarizing neuromuscular blocking agents such as pancuronium, d-tubocurarine, and atracurium.

Edrophonium is a cholinesterase inhibitor which can be used to reverse neuromuscular blockers. Fentanyl is an opioid and is actually an excellent choice to provide pain control and decrease the concentration of gas anesthetic needed in a procedure such as this one, but it would not accomplish the goal of preventing all motion, including respiration when necessary.

209
Q

You are presented a 6-month male DSH for hypersalivation and tremoring. The owner applied an over the counter topical flea medication this morning when she found 2 fleas on him. She did not bring the box from the flea medication. How should you immediately treat the cat?

Warm water enema, IV fluids, IV diazepam
IV dextrose to correct a likely hypoglycemia caused from the medication
Rinse mouth out, activated charcoal
Rinse off the topical medication and recommend flea collar only for ectoparasite control in the future
Bath to wash off the medication, IV fluids, IV methocarbamol

A

This cat is apparently suffering from Pyrethrin toxicity. Many cats are sensitive to flea medications, especially some over the counter varieties. The medication should be quickly washed off with a bath. The mouth should also be carefully rinsed depending on the status of the cat if ingestion is suspected. This often happens with cats that groom then lick the medication from the paws. IV catheter and fluids should be started immediately. Methocarbamol, a muscle relaxant, is the first best choice to stop the tremoring. If this is unsuccessful, valium can also be used, and if active seizures are occurring should be given immediately. Phenobarbital or Propofol drip can be given if seizures are refractory.

It is not likely that this medication has caused a hypoglycemia and dextrose is not indicated. A warm water enema would not be helpful as the toxicity is not occurring due to absorption of medication from the cat’s colon. Rinsing the mouth and activated charcoal may be indicated, but the initial treatments to stabilize the cat with the bath, fluids, and muscle relaxant are more critical on presentation.

Flea collars are often ineffective and many cats are sensitive to these as well. If the owner would like flea control, a veterinarian prescribed medication would be advised. Good choices for flea control include feline Frontline or Revolution.

210
Q

A 2-year old indoor, vaccinated, spayed female DSH presents with a 6-month history of lesions involving the feet. The owner reports that initially, only the posterior paws were involved but the lesions progressed to involve all four paws. The owner reports the cat did not improve after previous treatment with several antibiotics or a one week course of prednisone at 1 mg/kg.

On examination, you note that all of the metacarpal and metatarsal pads are soft, swollen and discolored as shown in the image. You note that the cat is uncomfortable when walking and occasionally licking the pads; one of the pads is fissured.

Bloodwork reveals a mild monocytosis and mild hyperglobulinemia. Which of the following tests will be most helpful in determining the diagnosis?

 Fungal culture of the lesion 
 Radiographs of the involved regions
 Abdominal ultrasound
 Serum electrophoresis
 Biopsy of the pad lesions
A

This is a case of plasma cell pododermatitis which is a relatively uncommon idiopathic disorder of cats. While the history and description of the lesions is consistent with this uncommon disorder, other differentials could include pemphigus foliaceus, pemphigus vulgaris, or lupus erythematosus. A chemical or physical trauma could cause lesions such as these but do not fit well with the history. Neoplasia, infection, or sterile pyogranulomas are possible but would be unlikely to affect all paws simultaneously. Eosinophilic granuloma complex is also possible.

Biopsy is likely the only way to differentiate between most of these possibilities. Because plasma cell pododermatitis is rare, it is unlikely that you would be asked about details of this disorder. With plasma cell pododermatitis, you would expect to see intense plasmacytic infiltration of the dermal tissue, potentially with other inflammatory cell types present due to secondary infection. The disease is not thought to have an infectious cause at this time. Treatment usually involves long courses (10 weeks) of Doxycycline. Approximately, 80% of cats will respond to Doxycycline. If a good response is not seen then systemic glucocorticoids can be tried. Surgical excision of the foot pad may be necessary in cases that fail to respond to medical management.

Although this question involves a rare disease that you probably would not be expected to know very much about, the question itself involves critical thinking and clinical judgement that you should be able to reason through if you can construct an appropriate list of differential diagnoses. The ability to read through the details of a complicated case and identify the information needed to decide what to do next will be an important skill on your board exam.

211
Q

A 9-year FS Domestic Long Hair presents for lethargy, weakness, and 2 seizures. The owner states that yesterday she gave the cat an enema she picked up from the drug store. The owner thought she had been constipated because she hadn’t defecated in several days and appeared uncomfortable.

She said it was a Fleet enema and that she had no problem administering it, but the cat vomited a few times afterward. Sassy is 5% dehydrated and obtunded on examination. She is having slight tremors. Your in-house laboratory won’t have bloodwork available for 2 hours. Which of the following treatments will most likely be indicated?

Potassium phosphate, a plain warm water enema, IV fluid therapy
Calcium gluconate, IV fluid therapy, phosphorus binders
Potassium phosphate, IV fluid therapy, methocarbamol
Insulin and dextrose, IV fluid therapy
Potassium phosphate, IV fluid therapy, diazepam

A

There are different Fleet enemas, some of which contain hypertonic sodium phosphate and are contraindicated in cats. Cats develop an electrolyte disturbance caused by the absorption of sodium and phosphate from the colon. This results in hypernatremia and hyperphosphatemia.

The high phosphorus leads to precipitation of serum calcium and thus hypocalcemia. This hypocalcemia can cause weakness, lead to shock, and cause muscle tremors or seizures. The treatment for this toxicity is to correct the electrolyte disturbance and correct the dehydration. IV fluid therapy and calcium gluconate are the initial treatments, and many times phosphorus binders are helpful to more quickly decrease the serum phosphorus.

Unless the cat is actively seizing, diazepam would not be indicated. The administration of calcium should help to stop the tremoring this cat is exhibiting. Potassium phosphate is clearly contraindicated, since the phosphorus is already too high. Insulin and dextrose are sometimes used in severe cases of hyperkalemia, which is not suspected in this case.

212
Q

An 8-year old male castrated cat presents to your clinic for a decreased appetite and slight weight loss over the past 2 months. Physical examination is unremarkable except for mild cranial organomegaly detected on abdominal palpation. You perform diagnostic tests and find the following abnormal results:

Complete blood count shows:
Hematocrit - 26% (30-45 %)

Glucose- 138 mg/dl (60-120 mg/dl) 
Globulin= 6.2 g/dl (2.6-5.1 g/dl) 
ALP- 85 IU/L (0-45 IU/L) 
ALT- 214 IU/L (25-97 IU/L) 
GGT- 9 IU/L (0-6 IU/L) 
Total bilirubin- 0.9 mg/dl (0-0.1 mg/dl) 

You perform an abdominal ultrasound and find that the liver appears subjectively enlarged. The echogenicity of the liver is normal and the gall bladder is normal in appearance. No other abnormalities are seen.

You perform an ultrasound guided liver biopsy. Histopathology indicates infiltration of lymphocytes and plasma cells but not neutrophils into portal areas but not into bile ducts.

With treatment, what is the cat’s prognosis?

Poor, mean survival is < 6 months
Fair, mean survival is about 1 year
Good, mean survival is greater than 2 years
Grave, mean survival is < 2 months

A

The case described is consistent with lymphocytic portal hepatitis. Clinically, this condition can appear similar to chronic cholangiohepatitis in terms of signalment, clinical signs and laboratory findings. The key to this diagnosis is the liver biopsy. Typical findings for lymphocytic portal hepatitis is infiltration of lymphocytes and plasma cells but not neutrophils into portal areas. This is in contrast to chronic cholangiohepatitis which typically has neutrophils in portal areas.

Chronic cholangiohepatitis carries a fair prognosis with about half of cats doing poorly (dead or euthanized within 3 months) and half of cats responding favorably to treatment with long term survival. For cats with lymphocytic portal hepatitis, although treatment can be challenging, the disease is very slowly progressive and the reported mean survival is approximately 3 years.

213
Q

A 5-month-old intact female mixed breed presents to your hospital for the progressive lesions seen on the image provided. The owner described the lesions as initially being papules and vesicles that eventually ruptured. Recently, the owner has also noticed that the muscles of the face appear to be atrophied. Muscle and skin biopsies were performed which came back supportive of a diagnosis of dermatomyositis. You start the patient on an immunosuppressive dose of steroids. Which other medication would be useful for the treatment of this condition?

Image used with permission, from Clinical Immunology of the Dog and Cat (Day), courtesy of Manson Publishing.
 Pentoxifylline 
 Meloxicam
 Hycodan
 Zinc
A

Dermatomyositis (DMS) is a relatively unusual condition that results in skin lesions as well as myositis. The skin lesions initially appear as papules and vesicles which then rupture, ulcerate, and may progress to crusty alopecic lesions. The myositis sets in a little later, and atrophy of the temporalis muscle is often seen. In more severe cases, the muscles of the distal extremities become involved, and megaesophagus may be observed. Since muscle changes usually lag, other diagnostic tests such as a skin scraping and cytology are performed first to rule out other common conditions. The condition has a variable prognosis, as the clinical signs may wax and wane. Medical management usually consists of pentoxifylline, prednisone, and cyclosporine. Prednisone and cyclosporine serve as immunomodulatory medications. Pentoxifylline is used to help improve blood flow.

Hycodan contains hydrocodone and is used as a cough suppressant. Meloxicam would be contraindicated in a patient on steroids. Although it looks similar to a zinc-responsive dermatosis, zinc would not be a treatment for DMS.

214
Q

Multiple Myeloma

A

This dog has signs and bloodwork findings that are consistent with but not specific for multiple myeloma. The hypercalcemia and hyperglobulinemia are both mild and are consistent with this diagnosis but would not be sufficient alone to jump to that conclusion. The key finding in these case is the radiographic evidence of multiple punctate lucencies including in the body of C4, several of the thoracic dorsal spinous processes, the left ilium, and the right and left humeral diaphyses. These lesions are strongly suggestive of multiple myeloma; however, definitive diagnosis of multiple myeloma requires satisfying at least two of the following criteria:

  1. Monoclonal gammopathy
  2. Radiographic evidence of osteolytic bone lesions (satisfied in this case)
  3. > 5% neoplastic cells or >10-20% plasma cells in the bone marrow
  4. Immunoglobulin light chain proteinuria (Bence-Jones proteinuria)

Therefore, the best answer in this question is to perform serum electrophoresis to assess monoclonal gammopathy and a bone marrow biopsy to look for neoplastic infiltration of plasma cells.

215
Q

A 2-year old MN DSH has recently been treated for a urethral obstruction. He went home last week from the hospital on an acidifying canned diet for this condition. The owner reports he is passing urine in moderate amounts, but he is still straining frequently. You re-examine the cat and find that the bladder is empty on palpation and the wall feels a little thickened. You are confident that the cat has not re-blocked and the cat’s bloodwork shows normal electrolytes and renal values. Which of the following medications may help the cat with this problem?

 Phenoxybenzamine 
 Prednisolone
 Methocarbamol
 Amitriptyline
 Phenylpropanolamine
A

This cat is likely suffering from hypertonicity of urethral muscle, which was incited from the recent obstruction and urinary catheter. This can cause spasms, which makes urinating painful and not easily controlled. Phenoxybenzamine can be used in this case to reduce internal urethral sphincter tone such that the cat may urinate more easily.

Methocarbamol is a muscle relaxant but would not directly help spasms of the urethra.

Prednisolone is not used to help reduce inflammation or spasms in the urethra and may predispose the cat to contracting a urinary infection, especially while his bladder and urethra are compromised.

Phenylpropanolamine is used to treat urinary incontinence from urethral hypotonicity most often in dogs and would be contraindicated in this case.

Amitriptyline is an anti-depressant medication that has been implicated as part of a treatment plan for cats with cystitis, although benefit has never been proven. Because cats with cystitis can flare up during times of stress, the amitriyptyline has been thought to help prevent this. This medication would not work to stop spasms in the urethra.

216
Q

A pregnant woman brings her cat to you to perform Toxoplasma gondii titers on it. The cat has no clinical signs. What would be appropriate to tell the woman?

A large percentage of cats are seropositive for Toxoplasma gondii, but are not necessarily shedding oocysts.
A single serum titer should be able to determine if the cat is likely shedding oocysts presently.
Toxoplasma oocysts take about 5 days to sporulate and become infective, so as long as the woman has the cat’s litterbox cleaned before 5 days have passed, she should not have to worry about being infected.
Cats seropositive for Toxoplasma gondii shed oocysts most of the time, so the woman should separate herself from the cat and its feces.

A

The correct answer is a large percentage of cats are seropositive for Toxoplasma gondii but are not necessarily shedding oocysts. Infected cats will usually only shed oocysts for 1-2 weeks when they are initially infected. They usually don’t shed again unless they become severely immune-compromised. Paired serum titers should be taken 1-2 weeks apart in order to determine the stage of infection. A rise in the consecutive titers would mean the cat was recently infected. Titers that aren’t significantly different can be interpreted as the cat having an old infection and is less likely to be actively shedding oocysts. Infectivity of the oocysts occurs with sporulation, which occurs within 1-5 days after shedding.